Combined Unit V

Ace your homework & exams now with Quizwiz!

A 55-year-old patient is preparing to start an exercise program. The health care provider wants 60% of maximum target heart rate. Calculate the heart rate that the nurse will add to the care plan as the target heart rate. Record answer as a whole number. _________ maximum heart rate

ANS: 99 Teach patients to calculate their maximum heart rate by subtracting their current age in years from 220 and then obtaining their target heart rate by taking 60% to 90% of the maximum, depending on their health care provider's recommendation. 220 − 55 = 165 × 0.6 = 99.

A patient has damage to the cerebellum. Which disorder is most important for the nurse to assess? a. Imbalance b. Hemiplegia c. Muscle sprain d. Lower extremity paralysis

ANS: A Damage to the cerebellum causes problems with balance, and motor impairment is directly related to the amount of destruction of the motor strip. A stroke can lead to hemiplegia. Direct trauma to the musculoskeletal system results in bruises, contusions, sprains, and fractures. A complete transection of the spinal cord can lead to lower extremity paralysis.

The patient weighs 450 lbs (204.5 kg) and reports shortness of breath with any exertion. The health care provider has recommended beginning an exercise program. The patient states that she can hardly get out of bed and just cannot do anything around the house. Which nursing diagnosis will the nurse add to the care plan? a. Activity intolerance related to excessive weight b. Impaired physical mobility related to bed rest c. Imbalanced nutrition: less than body requirements d. Impaired gas exchange related to shortness of breath

ANS: A In this case, activity intolerance is related to the patient's excessive weight. The patient is not on bed rest although claims that it is difficult to get out of bed, making this diagnosis inappropriate. Shortness of breath is a symptom, not a cause, of Impaired gas exchange, making this nursing diagnosis ineffective. The patient certainly has an imbalance of nutrition, but it is more than body requirements (obesity).

When assessing a patient's feet, the nurse notices that the toenails are thick and separated from the nail bed. What does the nurse most likely suspect is the cause of this condition? a. Fungi b. Friction c. Nail polish d. Nail polish remover

ANS: A Inflammatory lesions and fungus of the nail bed cause thickened, horny nails that separate from the nail bed. Ask women whether they frequently polish their nails and use polish remover because chemicals in these products cause excessive nail dryness. Friction and pressure from ill-fitting or loose shoes causes keratosis (corns). It is seen mainly on or between toes, over bony prominences.

A nurse is preparing to lavage a patient in the emergency department for an overdose. Which tube should the nurse obtain? a. Ewald b. Dobhoff c. Miller-Abbott d. Sengstaken-Blakemore

ANS: A Lavage is irrigation of the stomach in cases of active bleeding, poisoning, or gastric dilation. The types of tubes include Levin, Ewald, and Salem sump. Sengstaken-Blakemore is used for compression by internal application of pressure by means of inflated balloon to prevent internal esophageal or GI hemorrhage. Dobhoff is used for enteral feeding. Miller-Abbott is used for gastric decompression.

The nurse is completing an assessment of the patient's skin's integrity. Which assessment is the priority? a. Pressure points b. Breath sounds c. Bowel sounds d. Pulse points

ANS: A Observe pressure points such as bony prominences. The nurse continually assesses the skin for signs of ulcer development. Assessment for tissue pressure damage includes visual and tactile inspection of the skin. Assessment of pulses, breath sounds, and bowel sounds is part of a head-to-toe assessment and could influence the function of the body and ultimately skin integrity; however, this assessment is not a specific part or priority of a skin assessment.

Which initial intervention is most appropriate for a patient who has a new onset of chest pain? a. Reassess the patient. b. Notify the health care provider. c. Administer a prn medication for pain. d. Call radiology for a portable chest x-ray.

ANS: A Preparation for implementation ensures efficient, safe, and effective nursing care; the first activity is reassessment. The cause of the patient's chest pain is unknown, so the patient needs to be reassessed before pain medication is administered or a chest x-ray is obtained. The nurse then notifies the patient's health care provider of the patient's current condition in anticipation of receiving further orders. The patient's chest pain could be due to muscular injury or a pulmonary issue. The nurse needs to reassess first.

A nurse notices that a patient has a structural curvature of the spine associated with vertebral rotation. Which condition will the nurse most likely find documented in the patient's medical record? a. Scoliosis b. Arthritis c. Osteomalacia d. Osteogenesis

ANS: A Scoliosis is a structural curvature of the spine associated with vertebral rotation. Osteogenesis imperfecta is an inherited disorder that makes bones porous, short, bowed, and deformed. Osteomalacia is an uncommon metabolic disease characterized by inadequate and delayed mineralization, resulting in compact and spongy bone. Arthritis is an inflammatory joint disease characterized by inflammation or destruction of the synovial membrane and articular cartilage and by systemic signs of inflammation.

The nurse is caring for patients who need wound dressings. Match the type of dressing the nurse applies to its description. a. Absorbs drainage through the use of exudate absorbers in the dressing b. Very soothing to the patient and do not adhere to the wound bed c. Barrier to external fluids/bacteria but allows wound to "breathe" d. Manufactured from seaweed and comes in sheet and rope form e. Oldest and most common absorbent dressing 1. Gauze 2. Transparent 3. Hydrocolloid 4. Hydrogel 5. Calcium alginate

1.ANS:E 2.ANS:C 3.ANS:A 4.ANS:B 5.ANS:D

Upon assessment a nurse discovers postural abnormalities on several patients. Match the abnormalities to the findings the nurse observed. a. Lateral-S- or C-shaped spinal column with vertebral rotation b. Legs curved inward so knees come together as person walks c. One or both legs bent outward at knee d. Inclining of head to affected side e. Exaggeration of anterior convex curve of lumbar spine f. Increased convexity in curvature of thoracic spine 1. Lordosis 2. Kyphosis 3. Scoliosis 4. Genu valgum 5. Genu varum 6. Torticollis

1.ANS:E 2.ANS:F 3.ANS:A 4.ANS:B 5.ANS:C 6.ANS:D

A nurse is providing discharge teaching for a patient who is going home with a guaiac test. Which statement by the patient indicates the need for further education? a. "If I get a blue color that means the test is negative." b. "I should not get any urine on the stool I am testing." c. "If I eat red meat before my test, it could give me false results." d. "I should check with my doctor to stop taking aspirin before the test."

ANS: A A blue color indicates a positive guaiac, or presence of fecal occult blood; the patient needs more teaching to correct this misconception. Proper patient education is important for viable results. Be sure specimen is free of toilet paper and not contaminated with urine. The patient needs to avoid certain foods, like red meat, to rule out a false positive. While the health care provider should be consulted before asking a patient to stop any medication, if there are no contraindications, the patient should be instructed to stop taking aspirin, ibuprofen, naproxen or other nonsteroidal antiinflammatory drugs for 7 days because these could cause a false-positive test result.

A nurse is supervising the logrolling of a patient. To which patient is the nurse most likely providing care? a. A patient with neck surgery b. A patient with hypostatic pneumonia c. A patient with a total knee replacement d. A patient with a Stage IV pressure ulcer

ANS: A A nurse supervises and aids personnel when there is a health care provider's order to logroll a patient. Patients who have suffered from spinal cord injury or are recovering from neck, back, or spinal surgery often need to keep the spinal column in straight alignment to prevent further injury. Hypostatic pneumonia, total knee replacement, and Stage IV ulcers do not have to be logrolled.

A nurse is providing care to a group of patients. Which patient will the nurse see first? a. A patient with a hip replacement on prolonged bed rest reporting chest pain and dyspnea b. A bedridden patient who has a reddened area on the buttocks who needs to be turned c. A patient on bed rest who has renal calculi and needs to go to the bathroom d. A patient after knee surgery who needs range of motion exercises

ANS: A A patient on prolonged bed rest will be prone to deep vein thrombosis, which can lead to an embolus. An embolus can travel through the circulatory system to the lungs and impair circulation and oxygenation, resulting in tachycardia and shortness of breath. Venous emboli that travel to the lungs are sometimes life threatening. While the patient with a reddened area needs to be turned, a patient with renal calculi needing the restroom, and a patient needing range of motion, these are not as life threatening as the chest pain and dyspnea.

The standing orders for a patient include acetaminophen 650 mg every 4 hours prn for headache. After assessing the patient, the nurse identifies the need for headache relief and determines that the patient has not had acetaminophen in the past 4 hours. Which action will the nurse take next? a. Administer the acetaminophen. b. Notify the health care provider to obtain a verbal order. c. Direct the nursing assistive personnel to give the acetaminophen. d. Perform a pain assessment only after administering the acetaminophen.

ANS: A A standing order is a preprinted document containing orders for the conduct of routine therapies, monitoring guidelines, and/or diagnostic procedures for specific patients with identified clinical problems. The nurse will administer the medication. Notifying the health care provider is not necessary if a standing order exists. The nursing assistive personnel are not licensed to administer medications; therefore, medication administration should not be delegated to this person. A pain assessment should be performed before and after pain medication administration to assess the need for and effectiveness of the medication.

The nurse is caring for a patient who refuses to bathe in the morning. When asked why, the patient says "I always bathe in the evening." Which action by the nurse is best? a. Defer the bath until evening and pass on the information to the next shift. b. Tell the patient that daily morning baths are the "normal" routine. c. Explain the importance of maintaining morning hygiene practices. d. Cancel hygiene for the day and attempt again in the morning.

ANS: A Allow the patient to follow normal hygiene practices; change the bath to evening. Patients have individual preferences about when to perform hygiene and grooming care. Knowing the patient's personal preferences promotes individualized care for the patient. Hygiene care is never routine. Maintaining individual personal preferences is important unless new hygiene practices are indicated by an illness or condition. Cancelling hygiene and trying again is not an option since the nurse already knows the reason for refusal. Adapting practices to meet individual needs is required.

The patient has been brought to the emergency department following a motor vehicle accident. The patient is unresponsive. The driver's license states that glasses are needed to operate a motor vehicle, but no glasses were brought in with the patient. Which action should the nurse take next? a. Stand to the side of the patient's eye and observe the cornea. b. Conclude that the glasses were lost during the accident. c. Notify the ambulance personnel for missing glasses. d. Ask the patient where the glasses are.

ANS: A An important aspect of an eye examination is to determine if the patient wears contact lenses, especially in patients who are unresponsive. To determine whether a contact lens is present, stand to the side of the patient's eye and observe the cornea for the presence of a soft or rigid lens. It is also important to observe the sclera to detect the presence of a lens that has shifted off the cornea. An undetected lens causes severe corneal injury when left in place too long. Never assume that glasses were lost or were not worn. Contacting ambulance personnel takes time and cannot assume the glasses are missing. Asking the patient where the glasses are is inappropriate since the patient is unresponsive.

Which action indicates a nurse is using critical thinking for implementation of nursing care to patients? a. Determines whether an intervention is correct and appropriate for the given situation b. Reads over the steps and performs a procedure despite lack of clinical competency c. Establishes goals for a particular patient without assessment d. Evaluates the effectiveness of interventions

ANS: A As you implement interventions, use critical thinking to confirm whether the interventions are correct and still appropriate for a patient's clinical situation. You are responsible for having the necessary knowledge and clinical competency to perform interventions for your patients safely and effectively. The nurse needs to recognize the safety hazards of performing an intervention without clinical competency and seek assistance from another nurse. The nurse cannot evaluate interventions until they are implemented. Patients need ongoing assessment before establishing goals because patient conditions can change very rapidly.

The nurse is caring for a patient who has undergone surgery for a broken leg and has a cast in place. What should the nurse do to prevent skin impairment? a. Assess surfaces exposed to the edges of the cast for pressure areas. b. Keep the patient's blood pressure low to prevent overperfusion of tissue. c. Do not allow turning in bed because that may lead to redislocation of the leg. d. Restrict the patient's dietary intake to reduce the number of times on the bedpan.

ANS: A Assess surfaces exposed to casts, cloth restraints, bandages and dressings, tubing, or orthopedic devices. An external device applied to or around the skin exerts pressure or friction on the skin, leading to skin impairment. When restricted from moving, dependent body parts are exposed to pressure that reduces circulation to affected tissues, promoting pressure ulcers. Patients with limited caloric and protein intake develop impaired or delayed wound healing. Keeping the blood pressure artificially low may decrease arterial blood supply, leading to ischemia and breakdown.

The patient has been diagnosed with a spinal cord injury and needs to be repositioned using the logrolling technique. Which technique will the nurse use for logrolling? a. Use at least three people. b. Have the patient reach for the opposite side rail when turning. c. Move the top part of the patient's torso and then the bottom part. d. Do not use pillows after turning.

ANS: A At least three to four people are needed to perform this skill safely. Have the patient cross the arms on the chest to prevent injury to the arms. Move the patient as one unit in a smooth, continuous motion on the count of three. Gently lean the patient as a unit back toward pillows for support.

The patient is diagnosed with athlete's foot (tinea pedis). The patient says that he is relieved because it is only athlete's foot, and it can be treated easily. Which information should the nurse consider when formulating a response to the patient? a. Contagious with frequent recurrences b. Helpful to air-dry feet after bathing c. Treated with salicylic acid d. Caused by lice

ANS: A Athlete's foot spreads to other body parts, especially the hands. It is contagious and frequently recurs. Drying feet well after bathing and applying powder help prevent infection. It is caused by a fungus, not lice, and is treated with applications of griseofulvin, miconazole, or tolnaftate. Plantar wars are treated with salicylic acid or electrodesiccation.

The nurse is caring for a patient who has a wound drain with a collection device. The nurse notices that the collection device has a sudden decrease in drainage. Which action will the nurse take next? a. Call the health care provider; a blockage is present in the tubing. b. Chart the results on the intake and output flow sheet. c. Do nothing, as long as the evacuator is compressed. d. Remove the drain; a drain is no longer needed.

ANS: A Because a drainage system needs to be patent, look for drainage flow through the tubing, as well as around the tubing. A sudden decrease in drainage through the tubing may indicate a blocked drain, and you will need to notify the health care provider. The health care provider, not the nurse, determines the need for drain removal and removes drains. Charting the results on the intake and output flow sheet does not take care of the problem. The evacuator may be compressed even when a blockage is present.

The nurse is caring for a patient on the medical-surgical unit with a wound that has a drain and a dressing that needs changing. Which action should the nurse take first? a. Provide analgesic medications as ordered. b. Avoid accidentally removing the drain. c. Don sterile gloves. d. Gather supplies.

ANS: A Because removal of dressings is painful, if often helps to give an analgesic at least 30 minutes before exposing a wound and changing the dressing. The next sequence of events includes gathering supplies for the dressing change, donning gloves, and avoiding the accidental removal of the drain during the procedure.

The nurse is providing teaching to an immobilized patient with impaired skin integrity about diet. Which diet will the nurse recommend? a. High protein, high calorie b. High carbohydrate, low fat c. High vitamin A, high vitamin E d. Fluid restricted, bland

ANS: A Because the body needs protein to repair injured tissue and rebuild depleted protein stores, give the immobilized patient a high-protein, high-calorie diet. A high-carbohydrate, low-fat diet is not beneficial for an immobilized patient. Vitamins B and C are needed rather than A and E. Fluid restriction can be detrimental to the immobilized patient; this can lead to dehydration. A bland diet is not necessary for immobilized patients.

A patient has scaling of the scalp. Which term will the nurse use to report this finding to the oncoming staff? a. Dandruff b. Alopecia c. Pediculosis d. Xerostomia

ANS: A Dandruff is scaling of the scalp that is accompanied by itching. Pediculosis (lice infestation) resides on scalp attached to hair strands; eggs look like oval particles, similar to dandruff. Alopecia is hair loss or balding. Xerostomia is dry mouth.

The patient is being admitted to the neurological unit with a diagnosis of stroke. When will the nurse begin discharge planning? a. At the time of admission b. The day before the patient is to be discharged c. When outpatient therapy will no longer be needed d. As soon as the patient's discharge destination is known

ANS: A Discharge planning begins when a patient enters the health care system. In anticipation of the patient's discharge from an institution, the nurse makes appropriate referrals or consults a case manager or a discharge planner to ensure that the patient's needs are met at home. Referrals to home care or outpatient therapy are often needed. Planning the day before discharge, when outpatient therapy is no longer needed, and as soon as the discharge destination is known is too late.

The nurse is caring for an older-adult patient with Alzheimer's disease who is ambulatory but requires total assistance with activities of daily living (ADLs). The nurse notices that the patient is edentulous. Which area should the nurse assess? a. Assess oral cavity. b. Assess room for drafts. c. Assess ankles for edema. d. Assess for reduced sensations.

ANS: A Edentulous means without teeth; therefore, the nurse needs to assess the oral cavity. While older adults may want the room warmer and drafts should be avoided, this does not help with being edentulous. Edentulous does not mean the patient has edema. While older-adult patients can have reduced sensations, this is not the meaning of edentulous.

A nurse is teaching a health promotion class about isotonic exercises. Which types of exercises will the nurse give as examples? a. Swimming, jogging, and bicycling b. Tightening or tensing of muscles without moving body parts c. Quadriceps set exercises and contraction of the gluteal muscles d. Push-ups, hip lifting, pushing feet against a footboard on the bed

ANS: A Examples of isotonic exercises are walking, swimming, dance aerobics, jogging, bicycling, and moving arms and legs with light resistance. Isometric exercises involve tightening or tensing of muscles without moving body parts. Examples include quadriceps set exercises and contraction of the gluteal muscles. Examples of resistive isometric exercises are push-ups and hip lifting, as well as placing a footboard on the foot of the bed for patients to push against with their feet.

The patient is reporting an inability to clear nasal passages. Which action will the nurse take? a. Use gentle suction to prevent tissue damage. b. Instruct patient to blow nose forcefully to clear the passage. c. Place a dry washcloth under the nose to absorb secretions. d. Insert a cotton-tipped applicator to the back of the nose.

ANS: A Excessive nasal secretions can be removed using gentle suctioning. However, patients usually remove secretions from the nose by gentle blowing into a soft tissue. Caution the patient against harsh blowing that creates pressure capable of injuring the eardrum, the nasal mucosa, and even sensitive eye structures. If the patient is unable to remove nasal secretions, assist by using a wet washcloth or a cotton-tipped applicator moistened in water or saline. Never insert the applicator beyond the length of the cotton tip.

The debilitated patient is resisting attempts by the nurse to provide oral hygiene. Which action will the nurse take next? a. Insert an oral airway. b. Place the patient in a flat, supine position. c. Use undiluted hydrogen peroxide as a cleaner. d. Quickly proceed while not talking to the patient.

ANS: A If the patient is uncooperative, or is having difficulty keeping the mouth open, insert an oral airway. Insert it upside down, and then turn the airway sideways and over the tongue to keep the teeth apart. Do not use force. Position the patient on his or her side or turn the head to allow for drainage. Placing the patient in a flat, supine position could lead to aspiration. Hydrogen peroxide is irritating to mucosa. Even though the patient is debilitated, explain the steps of mouth care and the sensations that he or she will feel. Also tell the patient when the procedure is completed.

The nurse is bathing a patient and notices movement in the patient's hair. Which action will the nurse take? a. Use gloves to inspect the hair. b. Apply a lindane-based shampoo immediately. c. Shave the hair off of the patient's head. d. Ignore the movement and continue.

ANS: A In community health and home care settings, it is particularly important to inspect the hair for lice so appropriate hygienic treatment can be provided. If pediculosis capitis (head lice) is suspected, the nurse must protect self against self-infestations by handwashing and by using gloves or tongue blades to inspect the patient's hair. Suspicions cannot be ignored. Shaving hair off affected areas is the treatment for pediculosis pubis (crab lice) and is rarely used for head lice. Caution against use of products containing lindane because the ingredient is toxic and known to cause adverse reactions.

A nurse delegates a position change to a nursing assistive personnel. The nurse instructs the NAP to place the patient in the lateral position. Which finding by the nurse indicates a correct outcome? a. Patient is lying on side. b. Patient is lying on back. c. Patient is lying semiprone. d. Patient is lying on abdomen.

ANS: A In the side-lying (or lateral) position the patient rests on the side with the major portion of body weight on the dependent hip and shoulder. Patients in the supine position rest on their backs. Sims' position is semiprone. The patient in the prone position lies face or chest down on the abdomen.

The nurse is revising the care plan. In which order will the nurse perform the tasks, beginning with the first step? 1. Revise specific interventions. 2. Revise the assessment column. 3. Choose the evaluation method. 4. Delete irrelevant nursing diagnoses. a. 2, 4, 1, 3 b. 4, 2, 1, 3 c. 3, 4, 2, 1 d. 4, 2, 3, 1

ANS: A Modification of an existing written care plan includes four steps: 1. Revise data in the assessment column to reflect the patient's current status. Date any new data to inform other members of the health care team of the time that the change occurred. 2. Revise the nursing diagnoses. Delete nursing diagnoses that are no longer relevant and add and date any new diagnoses. Revise related factors and the patient's goals, outcomes, and priorities. Date any revisions. 3. Revise specific interventions that correspond to the new nursing diagnoses and goals. Be sure that revisions reflect the patient's present status. 4. Choose the method of evaluation for determining whether you achieved patient outcomes.

The nurse is working on a medical-surgical unit that has been participating in a research project associated with pressure ulcers. Which risk factor will the nurse assess for that predisposes a patient to pressure ulcer development? a. Decreased level of consciousness b. Adequate dietary intake c. Shortness of breath d. Muscular pain

ANS: A Patients who are confused or disoriented or who have changing levels of consciousness are unable to protect themselves. The patient may feel the pressure but may not understand what to do to relieve the discomfort or to communicate that he or she is feeling discomfort. Impaired sensory perception, impaired mobility, shear, friction, and moisture are other predisposing factors. Shortness of breath, muscular pain, and an adequate dietary intake are not included among the predisposing factors.

The nurse is caring for a patient who has diabetes mellitus and circulatory insufficiency, with peripheral neuropathy and urinary incontinence. On which areas does the nurse focus care? a. Decreased pain sensation and increased risk of skin impairment b. Decreased caloric intake and accelerated wound healing c. High risk for skin infection and low saliva pH level d. High risk for impaired venous return and dementia

ANS: A Patients with paralysis, circulatory insufficiency, or peripheral neuropathy (nerve damage) are unable to sense an injury to the skin (decreased pain sensation). The presence of urinary incontinence, circulatory insufficiency, and neuropathy can combine to result in breakdown, so the patient has an increased risk of skin impairment. While the patient may have decreased caloric intake, the patient will not have accelerated wound healing with circulatory insufficiency, neuropathy, and incontinence. While the patient is at high risk for skin infection, the low salivary pH level is not an issue. While the patient may have a high risk for impaired venous return from the circulatory insufficiency, there is no indication the patient has dementia.

The nurse is caring for a patient who needs to be placed in the prone position. Which action will the nurse take? a. Place pillow under the patient's abdomen after turning. b. Turn head toward one side with large, soft pillow. c. Position legs flat against bed. d. Raise head of bed to 45 degrees.

ANS: A Placing a pillow under the patient's abdomen after turning decreases hyperextension of lumbar vertebrae and strain on lower back; breathing may also be enhanced. Head is turned toward one side with a small pillow to reduce flexion or hyperextension of cervical vertebrae. Legs should be supported with pillows to elevate toes and prevent footdrop. Forty-five degrees is the position for Fowler's position; prone is on the stomach.

The nurse is preparing to move a patient to a wheelchair. Which action indicates the nurse is following recommendations for safe patient handling? a. Mentally reviews the transfer steps before beginning b. Uses own strength to transfer the patient c. Focuses solely on body mechanics d. Bases decisions on intuition

ANS: A Safe patient handling includes mentally reviewing the transfer steps before beginning the procedure to ensure both the patient's and your safety. Use the patient's strength when lifting, transferring, or moving when possible. Body mechanics alone do not protect the nurse from injury to the musculoskeletal system when moving, lifting, or transferring patients. After completing the assessment, nurses use an algorithm to guide decisions about safe patient handling.

A nurse is assigned most of the patients with pressure ulcers. The nurse leaves the pressure ulcer open to air and does not apply a dressing. To which patient did the nurse provide care? a. A patient with a clean Stage I b. A patient with a clean Stage II c. A patient with a clean Stage III d. A patient with a clean Stage IV

ANS: A Stage I intact pressure ulcers that resolve slowly without epidermal loss over 7 to 14 days do not require a dressing. A composite film, hydrocolloid, or hydrogel can be utilized on a clean Stage II. A hydrocolloid, hydrogel covered with foam, calcium alginate, and gauze can be utilized with a clean Stage III. Hydrogel covered with foam, calcium alginate, and gauze can be utilized with a clean Stage IV. An unstageable wound covered with eschar should utilize a dressing of adherent film or gauze with an ordered solution of enzymes.

A nurse is providing oral care to a patient with stomatitis. Which technique will the nurse use? a. Avoid commercial mouthwashes. b. Avoid normal saline rinses. c. Brush with a hard toothbrush. d. Brush with an alcohol-based toothpaste.

ANS: A Stomatitis causes burning, pain, and change in food and fluid tolerance. Advise patients to avoid alcohol and commercial mouthwash and stop smoking. When caring for patients with stomatitis, brush with a soft toothbrush and floss gently to prevent bleeding of the gums. In some cases, flossing needs to be temporarily omitted from oral care. Normal saline rinses (approximately 30 mL) on awaking in the morning, after each meal, and at bedtime help clean the oral cavity.

A patient is using laxatives three times daily to lose weight. After stopping laxative use, the patient has difficulty with constipation and wonders if laxatives should be taken again. Which information will the nurse share with the patient? a. Long-term laxative use causes the bowel to become less responsive to stimuli, and constipation may occur. b. Laxatives can cause trauma to the intestinal lining and scarring may result, leading to decreased peristalsis. c. Long-term use of emollient laxatives is effective for treatment of chronic constipation and may be useful in certain situations. d. Laxatives cause the body to become malnourished, so when the patient begins eating again, the body absorbs all of the food, and no waste products are produced.

ANS: A Teach patients about the potential harmful effects of overuse of laxatives, such as impaired bowel motility and decreased response to sensory stimulus. Make sure the patient understands that laxatives are not to be used long term for maintenance of bowel function. Increasing fluid and fiber intake can help with this problem. Laxatives do not cause scarring. Even if malnourished, the body will produce waste if any substance is consumed.

The nurse is intervening for a patient that has a risk for a urinary infection. Which direct care nursing intervention is most appropriate? a. Teaches proper handwashing technique b. Properly cleans the patient's toilet c. Transports urine specimen to the lab d. Informs the oncoming nurse during hand-off

ANS: A Teaching proper handwashing technique is a direct care nursing intervention. All the rest are indirect nursing care: cleaning the toilet, transporting specimens, and performing hand-off reports.

A patient is diagnosed with a bowel obstruction. Which type of tube is the best for the nurse to obtain for gastric decompression? a. Salem sump b. Small bore c. Levin d. 8 Fr

ANS: A The Salem sump tube is preferable for stomach decompression. The Salem sump tube has two lumina: one for removal of gastric contents and one to provide an air vent. When the main lumen of the sump tube is connected to suction, the air vent permits free, continuous drainage of secretions. While the Levin tube can be used for decompression, it is only a single-lumen tube with holes near the tip. Large-bore tubes, 12 Fr and above, are usually used for gastric decompression or removal of gastric secretions. Fine- or small-bore tubes are frequently used for medication administration and enteral feedings.

The nurse is starting an exercise program in a local community as a health promotion project. Which information will the nurse include in the teaching session? a. A cool-down period lasts about 5 to 10 minutes. b. The purpose of weight training is to bulk up muscles. c. Resistance training is appropriate for warm-up and cool-down periods. d. Aerobic exercise should be done 3 to 5 times per week for about 20 minutes.

ANS: A The cool-down period follows the exercise routine and usually lasts about 5 to 10 minutes. The purposes of weight training from a health perspective are to develop tone and strength and to simulate and maintain healthy bone. Stretching and flexibility exercises are ideal for warm-up and cool-down periods. The recommended frequency of aerobic exercise is 3 to 5 times per week or every other day for approximately 30 minutes.

A nurse is teaching a patient about the large intestine in elimination. In which order will the nurse list the structures, starting with the first portion? a. Cecum, ascending, transverse, descending, sigmoid, and rectum b. Ascending, transverse, descending, sigmoid, rectum, and cecum c. Cecum, sigmoid, ascending, transverse, descending, and rectum d. Ascending, transverse, descending, rectum, sigmoid, and cecum

ANS: A The large intestine is divided into the cecum, ascending colon, transverse colon, descending colon, sigmoid colon, and rectum. The large intestine is the primary organ of bowel elimination.

A nurse is developing an individualized plan of care for a patient. Which action is important for the nurse to take? a. Establish goals that are measurable and realistic. b. Set goals that are a little beyond the capabilities of the patient. c. Use the nurse's own judgment and not be swayed by family desires. d. Explain that without taking alignment risks, there can be no progress.

ANS: A The nurse must develop an individualized plan of care for each nursing diagnosis and must set goals that are individualized, realistic, and measurable. The nurse should set realistic expectations for care and should include the patient and family when possible. The goals focus on preventing problems or risks to body alignment and mobility.

A nurse is working in a facility that follows a comprehensive safe patient-handling program. Which finding will alert the nurse to intervene? a. Mechanical lifts are in a locked closet. b. Algorithms for patient handling are available. c. Ergonomic assessment protocols are being followed. d. A no-lift policy is in place with adherence by all staff.

ANS: A The nurse will follow up when lifts are not kept in convenient locations. Comprehensive safe patient-handling programs include the following elements: an ergonomics assessment protocol for health care environments, patient assessment criteria, algorithms for patient handling and movement, special equipment kept in convenient locations to help transfer patients, back injury resource nurses, an "after-action review" that allows the health care team to apply knowledge about moving patients safely in different settings, and a no-lift policy.

The nurse is devising a plan of care for a patient with the nursing diagnosis of Constipation related to opioid use. Which outcome will the nurse evaluate as successful for the patient to establish normal defecation? a. The patient reports eliminating a soft, formed stool. b. The patient has quit taking opioid pain medication. c. The patient's lower left quadrant is tender to the touch. d. The nurse hears bowel sounds in all four quadrants.

ANS: A The nurse's goal is for the patient to take opioid medication and to have normal bowel elimination. Normal stools are soft and formed. Ceasing pain medication is not a desired outcome for the patient. Tenderness in the left lower quadrant indicates constipation and does not indicate success. Bowel sounds indicate that the bowels are moving; however, they are not an indication of defecation.

A nurse is providing nursing care to patients after completing a care plan from nursing diagnoses. In which step of the nursing process is the nurse? a. Assessment b. Planning c. Implementation d. Evaluation

C Implementation, the fourth step of the nursing process, formally begins after a nurse develops a plan of care. With a care plan based on clear and relevant nursing diagnoses, a nurse initiates interventions that are designed to assist the patient in achieving the goals and expected outcomes needed to support or improve the patient's health status. The nurse gathers data during the assessment phase and mutually sets goals and prioritizes care during the planning phase. During the evaluation phase, the nurse determines the achievement of goals and effectiveness of interventions.

The nurse is caring for a patient with a healing Stage III pressure ulcer. Upon entering the room, the nurse notices an odor and observes a purulent discharge, along with increased redness at the wound site. What is the next best step for the nurse? a. Complete the head-to-toe assessment, including current treatment, vital signs, and laboratory results. b. Notify the health care provider by utilizing Situation, Background, Assessment, and Recommendation (SBAR). c. Consult the wound care nurse about the change in status and the potential for infection. d. Check with the charge nurse about the change in status and the potential for infection.

ANS: A The patient is showing signs and symptoms associated with infection in the wound. The nurse should complete the assessment: gather all data such as current treatment modalities, medications, vital signs including temperature, and laboratory results such as the most recent complete blood count or white cell count. The nurse can then notify the primary care provider and receive treatment orders for the patient. It is important to notify the charge nurse and consult the wound nurse on the patient's status and on any new orders.

Which nursing observation will indicate the patient is at risk for pressure ulcer formation? a. The patient has fecal incontinence. b. The patient ate two thirds of breakfast. c. The patient has a raised red rash on the right shin. d. The patient's capillary refill is less than 2 seconds.

ANS: A The presence and duration of moisture on the skin increase the risk of ulcer formation by making it susceptible to injury. Moisture can originate from wound drainage, excessive perspiration, and fecal or urinary incontinence. Bacteria and enzymes in the stool can enhance the opportunity for skin breakdown because the skin is moistened and softened, causing maceration. Eating a balanced diet is important for nutrition, but eating just two thirds of the meal does not indicate that the individual is at risk. A raised red rash on the leg again is a concern and can affect the integrity of the skin, but it is located on the shin, which is not a high-risk area for skin breakdown. Pressure can influence capillary refill, leading to skin breakdown, but this capillary response is within normal limits.

A nurse writes the following outcomes for a patient who has chronic obstructive pulmonary disease to improve activity level: Diastolic blood pressure will remain below 70 mm Hg with systolic below 130 mm Hg. Resting heart rate will range between 65 and 75. The last goal is that the patient will exercise 3 times a week. Which evaluative findings indicate successful goal achievement? (Select all that apply.) a. Resting heart rate 70 b. Blood pressure 126/64 c. Blood pressure 140/90 d. Reports doing stretching and flexibility exercises 2 times this week e. Reports doing resistive training 1 time and aerobics 2 times this week

ANS: A, B, E Compare actual outcomes with expected outcomes to determine the patient's health status and progression. Heart rate of 70 is between 65 and 75. Blood pressure 126/64 meets the goal. Did resistive training 1 time and aerobics 2 times equals exercising 3 times a week. Did stretching and flexibility exercises 2 times is below the 3 times a week. Blood pressure 140/90 is too high and does not meet the goal.

The nurse is caring for a patient who has had a recent stroke and is paralyzed on the left side. The patient has no respiratory or cardiac issues but cannot walk. The patient cannot button a shirt and cannot feed self due to being left-handed and becomes frustrated very easily. The patient has been eating very little and has lost 2 lbs. The patient asks the nurse, "How can I go home like this? I'm not getting better." Which health care team members will the nurse need to consult? (Select all that apply.) a. Dietitian b. Physical therapist c. Respiratory therapist d. Cardiac rehabilitation therapist e. Occupational therapist f. Psychologist

ANS: A, B, E, F Physical therapists are a resource for planning ROM or strengthening exercises, and occupational therapists are a resource for planning ADLs that patients need to modify or relearn. Because of the loss of 2 lbs and eating very little, a dietitian will also be helpful. Referral to a mental health advanced practice nurse, a licensed social worker, or a physiologist to assist with coping or other psychosocial issues is also wise. Because the patient exhibits good cardiac and respiratory function, respiratory therapy and cardiac rehabilitation probably are not needed at this time.

The nurse is caring for a patient with impaired physical mobility. Which potential complications will the nurse monitor for in this patient? (Select all that apply.) a. Footdrop b. Somnolence c. Hypostatic pneumonia d. Impaired skin integrity e. Increased socialization

ANS: A, C, D Immobility leads to complications such as hypostatic pneumonia. Other possible complications include footdrop and impaired skin integrity. Interruptions in the sleep-wake cycle and social isolation are more common complications than somnolence or increased socialization.

The nurse is caring for a patient who will have both a large abdominal bandage and an abdominal binder. Which actions will the nurse take before applying the bandage and binder? (Select all that apply.) a. Cover exposed wounds. b. Mark the sites of all abrasions. c. Assess the condition of current dressings. d. Inspect the skin for abrasions and edema. e. Cleanse the area with hydrogen peroxide. f. Assess the skin at underlying areas for circulatory impairment.

ANS: A, C, D, F Before applying a bandage or a binder, the nurse has several responsibilities. The nurse would need to inspect the skin for abrasions, edema, and discoloration or exposed wound edges. The nurse also is responsible for covering exposed wounds or open abrasions with a dressing and assessing the condition of underlying dressings and changing if soiled, as well as assessing the skin of underlying areas that will be distal to the bandage. This checks for signs of circulatory impairment, so that a comparison can be made after bandages are applied. Marking the sites of all abrasions is not necessary. Although it is important for the skin to be clean, and even though it may need to be cleaned with a noncytotoxic cleanser, cleansing with hydrogen peroxide can interfere with wound healing.

A nurse is performing an assessment on a patient who has not had a bowel movement in 3 days. The nurse will expect which other assessment finding? a. Hypoactive bowel sounds b. Increased fluid intake c. Soft tender abdomen d. Jaundice in sclera

ANS: A Three or more days with no bowel movement indicates hypomotility of the GI tract. Assessment findings would include hypoactive bowel sounds, a firm distended abdomen, and pain or discomfort upon palpation. Increased fluid intake would help the problem; a decreased intake can lead to constipation. Jaundice does not occur with constipation but can occur with liver disease.

The nurse is caring for a patient who has suffered a stroke and has residual mobility problems. The patient is at risk for skin impairment. Which initial actions should the nurse take to decrease this risk? a. Use gentle cleansers, and thoroughly dry the skin. b. Use therapeutic bed and mattress. c. Use absorbent pads and garments. d. Use products that hold moisture to the skin.

ANS: A Use cleansers with nonionic surfactants that are gentle to the skin. After you clean the skin, make sure that it is completely dry. Absorbent pads and garments are controversial and should be considered only when other alternatives have been exhausted. Depending on the needs of the patient, a specialty bed may be needed, but again, this does not provide the initial defense for skin breakdown. Use only products that wick moisture away from the patient's skin.

The nurse is caring for a patient who has multiple ticks on lower legs and body. What should the nurse do to rid the patient of ticks? a. Use blunt tweezers and pull upward with steady pressure. b. Burn the ticks with a match or small lighter. c. Allow the ticks to drop off by themselves. d. Apply miconazole and cover with plastic.

ANS: A Using blunt tweezers, grasp the tick as close to the head as possible and pull upward with even, steady pressure. Hold until the tick pulls out, usually for about 3 to 4 minutes. Save the tick in a plastic bag, and put it in the freezer if necessary to identify the type of tick. Because ticks transmit several diseases to people, they must be removed. Allowing them to drop off by themselves is not an option. Do not burn ticks off with a match or lighter. Miconazole is used to treat athlete's foot; it is a fungal medication. Covering ticks with plastic does not remove ticks.

A nurse is assessing the body alignment of a standing patient. Which finding will the nurse report as normal? a. When observed laterally, the spinal curves align in a reversed "S" pattern. b. When observed posteriorly, the hips and shoulders form an "S" pattern. c. The arms should be crossed over the chest or in the lap. d. The feet should be close together with toes pointed out.

ANS: A When the patient is observed laterally, the head is erect and the spinal curves are aligned in a reversed "S" pattern. When observed posteriorly, the shoulders and hips are straight and parallel. The arms hang comfortably at the sides. The feet are slightly apart to achieve a base of support, and the toes are pointed forward.

A nurse is providing hygiene care to a bariatric patient using chlorhexidine gluconate (CHG) wipes. Which actions will the nurse take? (Select all that apply.) a. Do not rinse. b. Clean under breasts. c. Inform that the skin will feel sticky. d. Dry thoroughly between skin folds. e. Use two wipes for each area of the body.

ANS: A, B, C CHG wipes are easy to use and accessible for older patients and bariatric patients, offering a no-rinse or -drying procedure. For a bariatric patient or a patient who is diaphoretic, provide special attention to body areas such as beneath the woman's breasts, in the groin, skin folds, and perineal area, where moisture collects and irritates skin surfaces. Use wipes as directed on package—one wipe per each area of the body. CHG can leave the skin feeling sticky. If patients complain about its use, you need to explain their vulnerability to infection and how CHG helps reduce occurrence of health care-associated infection.

A nurse is teaching a health class about colorectal cancer. Which information should the nurse include in the teaching session? (Select all that apply.) a. A risk factor is smoking. b. A risk factor is high intake of animal fats or red meat. c. A warning sign is rectal bleeding. d. A warning sign is a sense of incomplete evacuation. e. Screening with a colonoscopy is every 5 years, starting at age 50. f. Screening with flexible sigmoidoscopy is every 10 years, starting at age 50.

ANS: A, B, C, D Risk factors for colorectal cancer are a diet high in animal fats or red meat and low intake of fruits and vegetables; smoking and heavy alcohol consumption are also risk factors. Warning signs are change in bowel habits, rectal bleeding, a sensation of incomplete evacuation, and unexplained abdominal or back pain. A flexible sigmoidoscopy is every 5 years, starting at age 50, while a colonoscopy is every 10 years, starting at age 50.

A nurse is providing nursing care to a group of patients. Which actions are direct care interventions? (Select all that apply.) a. Ambulating a patient b. Inserting a feeding tube c. Performing resuscitation d. Documenting wound care e. Teaching about medications

ANS: A, B, C, E All of the interventions listed (ambulating, inserting a feeding tube, performing resuscitation, and teaching) are direct care interventions involving patient and nurse interaction, except documenting wound care. Documenting wound care is an example of an indirect intervention.

The nurse is completing a skin assessment on a medical-surgical patient. Which nursing assessment questions should be included in a skin integrity assessment? (Select all that apply.) a. "Can you easily change your position?" b. "Do you have sensitivity to heat or cold?" c. "How often do you need to use the toilet?" d. "What medications do you take?" e. "Is movement painful?" f. "Have you ever fallen?"

ANS: A, B, C, E Changing positions is important for decreasing the pressure associated with long periods of time in the same position. If the patient is able to feel heat or cold and is mobile, she can protect herself by withdrawing from the source. Knowing toileting habits and any potential for incontinence is important because urine and feces in contact with the skin for long periods can increase skin breakdown. Knowing whether the patient has problems with painful movement will alert the nurse to any potential for decreased movement and increased risk for skin breakdown. Medications and falling are safety risk questions.

Which interventions are appropriate for a patient with diabetes and poor wound healing? (Select all that apply.) a. Perform dressing changes twice a day as ordered. b. Teach the patient about signs and symptoms of infection. c. Instruct the family about how to perform dressing changes. d. Gently refocus patient from discussing body image changes. e. Administer medications to control the patient's blood sugar as ordered.

ANS: A, B, C, E Nursing priorities include interventions directed at enhancing wound healing. Teaching the patient about signs and symptoms of infection will help the patient identify signs of appropriate wound healing and know when the need for calling the health care provider arises. Performing dressing changes, controlling blood sugars through administration of medications, and instructing the family in dressing changes all contribute to wound healing. As long as a patient is stable and alert, it is appropriate to allow family to assist with care. The patient should be allowed to discuss body image changes.

A nurse is assessing activity tolerance of a patient. Which areas will the nurse assess? (Select all that apply.) a. Skeletal abnormalities b. Emotional factors c. Pregnancy status d. Race e. Age

ANS: A, B, C, E Physiological, emotional, and developmental factors (age) influence the patient's activity tolerance. Factors influencing activity tolerance include physiological factors such as skeletal abnormalities, emotional factors such as anxiety/depression, developmental factors such as age and gender, and pregnancy status. Race is not a factor because people of all races are faced with similar factors that affect their activity tolerance.

A nurse is preparing to carry out interventions. Which resources will the nurse make sure are available? (Select all that apply.) a. Equipment b. Safe environment c. Confidence d. Assistive personnel e. Creativity

ANS: A, B, D A nurse will organize time and resources in preparation for implementing nursing care. Most nursing procedures require some equipment or supplies. Before performing an intervention, decide which supplies you need and determine their availability. Patient care staff (assistive personnel) work together as patients' needs demand it. A patient's care environment needs to be safe and conducive to implementing therapies. Confidence and creativity are needed to provide safe and effective patient care; however, these are critical thinking attitudes, not resources.

Which patients will the nurse determine are in need of perineal care? (Select all that apply.) a. A patient with rectal and genital surgical dressings b. A patient with urinary and fecal incontinence c. A circumcised male who is ambulatory d. A patient who has an indwelling catheter e. A bariatric patient

ANS: A, B, D, E Patients most in need of perineal care include those at greatest risk for acquiring an infection (e.g., uncircumcised males, patients who have indwelling urinary catheters, or those who are recovering from rectal or genital surgery or childbirth). A patient with urinary and bowel incontinence needs perineal cleaning with each episode of soiling. Bariatric patients need special attention to body areas such as skin folds and the perineal area. In addition, women who are having a menstrual period require perineal care. Circumcised males are not at high risk for acquiring infection, and ambulatory patients can usually provide perineal self-care.

A nurse is preparing a bowel training program for a patient. Which actions will the nurse take? (Select all that apply.) a. Record times when the patient is incontinent. b. Help the patient to the toilet at the designated time. c. Lean backward on the hips while sitting on the toilet. d. Maintain normal exercise within the patient's physical ability. e. Apply pressure with hands over the abdomen, and strain while pushing. f. Choose a time based on the patient's pattern to initiate defecation-control measures.

ANS: A, B, D, F A successful program includes the following: Assessing the normal elimination pattern and recording times when the patient is incontinent. Choosing a time based on the patient's pattern to initiate defecation-control measures. Maintaining normal exercise within the patient's physical ability. Helping the patient to the toilet at the designated time. Offering a hot drink (hot tea) or fruit juice (prune juice) (or whatever fluids normally stimulate peristalsis for the patient) before the defecation time. Instructing the patient to lean forward at the hips while sitting on the toilet, apply manual pressure with the hands over the abdomen, and bear down but do not strain to stimulate colon emptying.

The patient is eager to begin an exercise program with a 2-mile jog. The nurse instructs the patient to warm up. The patient does not want to waste time with a "warm-up." Which information will the nurse share with the patient? a. The warm-up in this case can be done after the 2-mile jog. b. The warm-up prepares the body and decreases the potential for injury. c. The warm-up allows the body to readjust gradually to baseline functioning. d. The warm-up should be performed with high intensity to prepare for the coming challenge.

ANS: B The warm-up activity prepares the body for activity and decreases the potential for injury and should not be omitted. It usually lasts about 5 to 10 minutes and may include stretching, calisthenics, and/or aerobic activity performed at a lower intensity. The warm-up is before the exercise, while the cool-down period is after the exercise. The cool-down, not the warm-up, allows the body to readjust gradually to baseline functioning and provides an opportunity to combine movement such as stretching with relaxation-enhancing mind-body awareness. The warm-up should not be a high-intensity workout.

The nurse is admitting an older patient from a nursing home. During the assessment, the nurse notes a shallow open reddish, pink ulcer without slough on the right heel of the patient. How will the nurse stage this pressure ulcer? a. Stage I b. Stage II c. Stage III d. Stage IV

ANS: B This would be a Stage II pressure ulcer because it presents as partial-thickness skin loss involving epidermis and dermis. The ulcer presents clinically as an abrasion, blister, or shallow crater. Stage I is intact skin with nonblanchable redness over a bony prominence. With a Stage III pressure ulcer, subcutaneous fat may be visible, but bone, tendon, and muscles are not exposed. Stage IV involves full-thickness tissue loss with exposed bone, tendon, or muscle.

The nurse is teaching a patient about contact lens care. Which instructions will the nurse include in the teaching session? a. Use tap water to clean soft lenses. b. Wash and rinse lens storage case daily. c. Reuse storage solution for up to a week. d. Keep the lenses is a cool dry place when not being used.

ANS: B Thoroughly wash and rinse lens storage case on a daily basis. Clean periodically with soap or liquid detergent, rinse thoroughly with warm water, and air-dry. Do not use tap water to clean soft lenses. Lenses should be kept moist or wet when not worn. Use fresh solution daily when storing and disinfecting lenses.

The nurse is preparing to provide a complete bed bath to an unconscious patient. The nurse decides to use a bag bath. In which order will the nurse clean the body, starting with the first area? 1. Neck, shoulders, and chest 2. Abdomen and groin/perineum 3. Legs, feet, and web spaces 4. Back of neck, back, and then buttocks 5. Both arms, both hands, web spaces, and axilla a. 5, 1, 2, 3, 4 b. 1, 5, 2, 3, 4 c. 1, 5, 2, 4, 3 d. 5, 1, 2, 4, 3

ANS: B Use all six chlorhexidene gluconate (CHG) cloths in the following order: 1. Cloth 1: Neck, shoulders, and chest 2. Cloth 2: Both arms, both hands, web spaces, and axilla 3. Cloth 3: Abdomen and then groin/perineum 4. Cloth 4: Right leg, right foot, and web spaces 5. Cloth 5: Left leg, left foot, and web spaces 6. Cloth 6: Back of neck, back, and then buttocks

The nurse is preparing to transfer an uncooperative patient who does not have upper body strength. Which piece of equipment will be best for the nurses to obtain? a. Drawsheet b. Full body sling c. Overhead trapeze d. Friction-reducing slide sheet

ANS: B Using a mechanical lift and full body sling to transfer an uncooperative patient who can bear partial weight or a patient who cannot bear weight and is either uncooperative or does not have upper body strength to move from bed to chair prevents musculoskeletal injuries to health care workers. The nurse should not attempt to move the patient with a drawsheet. The patient does not have upper body strength so an overhead trapeze is not appropriate. A friction-reducing slide sheet that minimizes shearing forces is not as effective as a full body sling.

The nurse has attempted to administer a tap water enema for a patient with fecal impaction with no success. The fecal mass is too large for the patient to pass voluntarily. Which is the next priority nursing action? a. Preparing the patient for a second tap water enema b. Obtaining an order for digital removal of stool c. Positioning the patient on the left side d. Inserting a rectal tube

ANS: B When enemas are not successful, digital removal of the stool may be necessary to break up pieces of the stool or to stimulate the anus to defecate. Tap water enemas should not be repeated because of risk of fluid imbalance. Positioning the patient on the left side does not promote defecation. A rectal tube is indicated for a patient with liquid stool incontinence or flatus but would not be applicable or effective for this patient.

The nurse is preparing to lift a patient. Which action will the nurse take first? a. Position a drawsheet under the patient. b. Assess weight and determine assistance needs. c. Delegate the task to a nursing assistive personnel. d. Attempt to manually lift the patient alone before asking for assistance.

ANS: B When lifting, assess the weight you will lift, and determine the assistance you will need. The nurse has to assess before positioning a drawsheet or delegating the task. Manual lifting is the last resort, and it is used when the task at hand does not involve lifting most or all of the patient's weight; most facilities have a no-lift policy.

The nurse is caring for a patient who is at risk for skin impairment. The patient is able to sit up in a chair. The nurse includes this intervention in the plan of care. How long should the nurse schedule the patient to sit in the chair? a. At least 3 hours b. Less than 2 hours c. No longer than 30 minutes d. As long as the patient remains comfortable

ANS: B When patients are able to sit up in a chair, make sure to limit the amount of time to 2 hours or less. The chair sitting time should be individualized. In the sitting position, pressure on the ischial tuberosities is greater than in a supine position. Utilize foam, gel, or an air cushion to distribute weight. Sitting for longer than 2 hours can increase the chance of ischemia.

The nurse is caring for a patient who is immobile and is at risk for skin impairment. The plan of care includes turning the patient. Which is the best method for repositioning the patient? a. Place the patient in a 30-degree supine position. b. Utilize a transfer device to lift the patient. c. Elevate the head of the bed 45 degrees. d. Slide the patient into the new position.

ANS: B When repositioning the patient, obtain assistance and utilize a transfer device to lift rather than drag the patient. Sliding the patient into the new position will increase friction. The patient should be placed in a 30-degree lateral position, not a supine position. The head of the bed should be elevated less than 30 degrees to prevent pressure ulcer development from shearing forces.

A patient uses an in-the-canal hearing aid. Which assessment is a priority? a. Eyeglass usage b. Cerumen buildup c. Type of physical exercise d. Excessive moisture problems

ANS: B With this type of model (in-the-canal), cerumen tends to plug this model more than others. There are three popular types of hearing aids. An in-the-canal (ITC) aid is the newest, smallest, and least visible and fits entirely in the ear canal. It has cosmetic appeal, is easy to manipulate and place in the ear, and does not interfere with wearing eyeglasses or using the telephone, and the patient can wear it during most physical exercise. An in-the-ear aid (ITE, or intra-aural) is more noticeable than the ITC aid and is not for people with moisture or skin problems in the ear canal. The larger size of this type of aid (behind-the-ear, BTE, or post-aural) can make use of eyeglasses and phones difficult; it is more difficult to keep in place during physical exercise.

The nurse is caring for a patient with cognitive impairments. Which actions will the nurse take during AM care? (Select all that apply.) a. Administer ordered analgesic 1 hour before bath time. b. Increase the frequency of skin assessment. c. Reduce triggers in the environment. d. Keep the room temperature cool. e. Be as quick as possible.

ANS: B, C If a patient is physically dependent or cognitively impaired, increase the frequency of skin assessment. Adapt your bathing procedures and the environment to reduce the triggers. For example, administer any ordered analgesic 30 minutes before a bath and be gentle in your approach. Keep the patient's body as warm as possible with warm towels and be sure the room temperature is comfortable.

The nurse is caring for a patient with potential skin breakdown. Which components will the nurse include in the skin assessment? (Select all that apply.) a. Vision b. Hyperemia c. Induration d. Blanching e. Temperature of skin

ANS: B, C, D, E Assessment of the skin includes both visual and tactile inspection. Assess for hyperemia and palpate for blanching or nonblaching. Early signs of skin damage include induration, bogginess (less-than-normal stiffness), and increased warmth at the injury site compared to nearby areas. Changes in temperature can indicate changes in blood flow to that area of the skin. Vision is not included in the skin assessment.

The nurse is updating the plan of care for a patient with impaired skin integrity. Which findings indicate achievement of goals and outcomes? (Select all that apply.) a. The patient's expectations are not being met. b. Skin is intact with no redness or swelling. c. Nonblanchable erythema is absent. d. No injuries to the skin and tissues are evident. e. Granulation tissue is present.

ANS: B, C, D, E Optimal outcomes are to prevent injury to skin and tissues, reduce injury to skin, reduce injury to underlying tissues, and restore skin integrity. Skin intact, nonblanchable erythema absent, no injuries, and presence of granulation tissue are all findings indicating achievement of goals and outcomes. The patient's expectations not being met indicates no progression toward goals/outcomes.

A nurse is preparing to move a patient who is able to assist. Which principles will the nurse consider when planning for safe patient handling? (Select all that apply.) a. Keep the body's center of gravity high. b. Face the direction of the movement. c. Keep the base of support narrow. d. Use the under-axilla technique. e. Use proper body mechanics. f. Use arms and legs.

ANS: B, E, F When a patient is able to assist, remember the following principles: The wider the base of support, the greater the stability of the nurse; the lower the center of gravity, the greater the stability of the nurse; facing the direction of movement prevents abnormal twisting of the spine. The use of assistive equipment and continued use of proper body mechanics significantly reduces the risk of musculoskeletal injuries. Use arms and legs (not back) because the leg muscles are stronger, larger muscles capable of greater work without injury. The under-axilla technique is physically stressful for nurses and uncomfortable for patients.

Which nutritional instruction is a priority for the nurse to advise a patient about with an ileostomy? a. Keep fiber low. b. Eat large meals. c. Increase fluid intake. d. Chew food thoroughly.

ANS: C Patients with ileostomies will digest their food completely but will lose both fluid and salt through their stoma and will need to be sure to replace this to avoid dehydration. A good reminder for patients is to encourage drinking an 8-ounce glass of fluid when they empty their pouch. This helps patients to remember that they have greater fluid needs than they did before having an ileostomy. A low-fiber diet is not necessary. Eating large meals is not advised. While chewing food thoroughly is correct, it is not the priority; liquid is the priority.

A nurse is reviewing a patient's care plan. Which information will the nurse identify as a nursing intervention? a. The patient will ambulate in the hallway twice this shift using crutches correctly. b. Impaired physical mobility related to inability to bear weight on right leg. c. Provide assistance while the patient walks in the hallway twice this shift with crutches. d. The patient is unable to bear weight on right lower extremity.

ANS: C Providing assistance to a patient who is ambulating is a nursing intervention. The statement, "The patient will ambulate in the hallway twice this shift using crutches correctly" is a patient outcome. Impaired physical mobility is a nursing diagnosis. The statement that the patient is unable to bear weight and ambulate can be included with assessment data and is a defining characteristic for the diagnosis of Impaired physical mobility.

A nurse is preparing to assess a patient for orthostatic hypotension. Which piece of equipment will the nurse obtain to assess for this condition? a. Thermometer b. Elastic stockings c. Blood pressure cuff d. Sequential compression devices

ANS: C A blood pressure cuff is needed. Orthostatic hypotension is a drop of blood pressure greater than 20 mm Hg in systolic pressure or 10 mm Hg in diastolic pressure and symptoms of dizziness, light-headedness, nausea, tachycardia, pallor, or fainting when the patient changes from the supine to standing position. A thermometer is used to assess for fever. Elastic stockings and sequential compression devices are used to prevent thrombus.

The nurse administers a cathartic to a patient. Which finding helps the nurse determine that the cathartic has a therapeutic effect? a. Reports decreased diarrhea. b. Experiences pain relief. c. Has a bowel movement. d. Passes flatulence.

ANS: C A cathartic is a laxative that stimulates a bowel movement. It would be effective if the patient experiences a bowel movement. The other options are not outcomes of administration of a cathartic. An antidiarrheal will provide relief from diarrhea. Pain medications will provide pain relief. Carminative enemas provide relief from gaseous distention (flatulence).

A patient is receiving a neomycin solution enema. Which primary goal is the nurse trying to achieve? a. Prevent gaseous distention b. Prevent constipation c. Prevent colon infection d. Prevent lower bowel inflammation

ANS: C A medicated enema is a neomycin solution, i.e., an antibiotic used to reduce bacteria in the colon before bowel surgery. Carminative enemas provide relief from gaseous distention. Bulk forming, emollient (wetting), and osmotic laxatives and cathartics help prevent constipation or treat constipation. An enema containing steroid medication may be used for acute inflammation in the lower colon.

A self-sufficient bedridden patient is unable to reach all body parts. Which type of bath will the nurse assign to the nursing assistive personnel? a. Bag bath b. Sponge bath c. Partial bed bath d. Complete bed bath

ANS: C A partial bath consists of washing body parts that the patient cannot reach, including the back, and providing a backrub. Dependent patients in need of partial hygiene or self-sufficient bedridden patients who are unable to reach all body parts receive a partial bed bath. Complete bed baths are administered to totally dependent patients in bed. The bag bath contains several soft, nonwoven cotton cloths that are premoistened in a solution of no-rinse surfactant cleanser and emollient. The sponge bath involves bathing from a bath basin or a sink with the patient sitting in a chair.

The nurse is caring for a patient who had a colostomy placed yesterday. The nurse should report which assessment finding immediately? a. Stoma is protruding from the abdomen. b. Stoma is flush with the skin. c. Stoma is purple. d. Stoma is moist.

ANS: C A purple stoma may indicate strangulation/necrosis or poor circulation to the stoma and may require surgical intervention. A stoma should be reddish-pink and moist in appearance. It can be flush with the skin, or it can protrude.

The nurse is caring for a patient with a spinal cord injury and notices that the patient's hips have a tendency to rotate externally when the patient is supine. Which device will the nurse use to help prevent injury secondary to this rotation? a. Hand rolls b. A trapeze bar c. A trochanter roll d. Hand-wrist splints

ANS: C A trochanter roll prevents external rotation of the hips when the patient is in a supine position. Hand rolls maintain the thumb in slight adduction and in opposition to the fingers. Hand-wrist splints are individually molded for the patient to maintain proper alignment of the thumb and the wrist. The trapeze bar is a triangular device that hangs down from a securely fastened overhead bar that is attached to the bedframe. It allows the patient to pull with the upper extremities to raise the trunk off the bed, to assist in transfer from bed to wheelchair, or to perform upper arm exercises.

The nurse is performing a fecal occult blood test. Which action should the nurse take? a. Test the quality control section before testing the stool specimens. b. Apply liberal amounts of stool to the guaiac paper. c. Report a positive finding to the provider. d. Don sterile disposable gloves.

ANS: C Abnormal findings such as a positive test (turns blue) should be reported to the provider. A fecal occult blood test is a clean procedure; sterile gloves are not needed. A thin specimen smear is all that is required. The quality control section should be developed after it is determined whether the sample is positive or negative.

The nurse will irrigate a patient's nasogastric (NG) tube. Which action should the nurse take? a. Instill solution into pigtail slowly. b. Check placement after instillation of solution. c. Immediately aspirate after instilling fluid. d. Prepare 60 mL of tap water into Asepto syringe.

ANS: C After instilling saline, immediately aspirate or pull back slowly on syringe to withdraw fluid. Do not introduce saline through blue "pigtail" air vent of Salem sump tube. Checking placement before instillation of normal saline prevents accidental entrance of irrigating solution into lungs. Draw up 30 mL of normal saline into Asepto syringe to minimize loss of electrolytes from stomach fluids.

The nurse is caring for a patient with a Stage IV pressure ulcer. Which nursing diagnosis does the nurse add to the care plan? a. Readiness for enhanced nutrition b. Impaired physical mobility c. Impaired skin integrity d. Chronic pain

ANS: C After the assessment is completed and the information that the patient has a Stage IV pressure ulcer is gathered, a diagnosis of Impaired skin integrity is selected. Readiness for enhanced nutrition would be selected for an individual with an adequate diet that could be improved. Impaired physical mobility and Chronic pain do not support the current data in the question.

The nurse is caring for a patient who requires a complex dressing change. While in the patient's room, the nurse decides to change the dressing. Which action will the nurse take just before changing the dressing? a. Gathers and organizes needed supplies b. Decides on goals and outcomes for the patient c. Assesses the patient's readiness for the procedure d. Calls for assistance from another nursing staff member

ANS: C Always be sure a patient is physically and psychologically ready for any interventions or procedures. After determining the patient's readiness for the dressing change, the nurse gathers needed supplies. The nurse establishes goals and outcomes before intervening. The nurse needs to ask another staff member to help if necessary after determining readiness of the patient.

A nurse is providing care to a group of patients. Which patient will the nurse see first? a. A child about to receive a normal saline enema b. A teenager about to receive loperamide for diarrhea c. An older patient with glaucoma about to receive an enema d. A middle-aged patient with myocardial infarction about to receive docusate sodium

ANS: C An enema is contradicted in a patient with glaucoma; this patient should be seen first. All the rest are expected. A child can receive normal saline enemas since they are isotonic. Loperamide, an antidiarrheal, is given for diarrhea. Docusate sodium is given to soften stool for patients with myocardial infarction to prevent straining.

The nurse needs to move a patient up in bed using a drawsheet. The nurse has another nurse helping. In which order will the nurses perform the steps, beginning with the first one? 1. Grasp the drawsheet firmly near the patient. 2. Move the patient and drawsheet to the desired position. 3. Position one nurse at each side of the bed. 4. Place the drawsheet under the patient from shoulder to thigh. 5. Place your feet apart with a forward-backward stance. 6. Flex knees and hips and on count of three shift weight from the front to back leg. a. 1, 4, 5, 6, 3, 2 b. 4, 1, 3, 5, 6, 2 c. 3, 4, 1, 5, 6, 2 d. 5, 6, 3, 1, 4, 2

ANS: C Assisting a patient up in bed with a drawsheet (two or three nurses): (1) Place the patient supine with the head of the bed flat. A nurse stands on each side of the bed. (2) Remove the pillow from under the patient's head and shoulders and place it at the head of the bed. (3) Turn the patient side to side to place the drawsheet under the patient, extending it from shoulders to thighs. (4) Return the patient to the supine position. (5) Fanfold the drawsheet on both sides, with each nurse grasping firmly near the patient. (6) Nurses place their feet apart with a forward-backward stance. Nurses should flex knees and hips. On the count of three, nurses should shift their weight from front to back leg and move the patient and drawsheet to the desired position in the bed.

The nurse is working on an orthopedic rehabilitation unit that requires lifting and positioning of patients. Which personal injury will the nurse most likely try to prevent? a. Arm b. Hip c. Back d. Ankle

ANS: C Back injuries are often the direct result of improper lifting and bending. The most common back injury is strain on the lumbar muscle group. While arm, hip, and ankle can occur, they are not as common as back.

A newly admitted patient who is morbidly obese asks the nurse for assistance to the bathroom for the first time. Which action should the nurse take initially? a. Ask for at least two other assistive personnel to come to the room. b. Medicate the patient to alleviate discomfort while ambulating. c. Review the patient's activity orders. d. Offer the patient a walker.

ANS: C Before beginning care, review the plan to determine the need for assistance and the type required. Before intervening, the nurse must check the patient's orders. For example, if the patient is on bed rest, the nurse will need to explain the use of a bedpan rather than helping the patient get out of bed to go to the bathroom. Asking for assistive personnel is appropriate after making sure the patient can get out of bed. If the patient is obese, the nurse will likely need assistance in getting the patient to the bathroom. Medicating the patient before checking the orders is not advised in this situation. Before medicating for pain, the nurse needs to perform a pain assessment. Offering the patient a walker is a premature intervention until the orders are verified.

The nurse is completing an assessment on a patient who has a Stage IV pressure ulcer. The wound is odorous, and a drain is currently in place. Which statement by the patient indicates issues with self-concept? a. "I am so weak and tired. I want to feel better." b. "I am thinking I will be ready to go home early next week." c. "I am ready for my bath and linen change right now since this is awful." d. "I am hoping there will be something good for dinner tonight."

ANS: C Body image changes can influence self-concept. The wound is odorous, and a drain is in place. The patient who is asking for a bath and change in linens and states that this is awful gives you a clue that he or she may be concerned about the smell in the room. Factors that affect the patient's perception of the wound include the presence of scars, drains, odor from drainage, and temporary or permanent prosthetic devices. The patient's stating that he or she wants to feel better, talking about going home, and caring about what is for dinner could be interpreted as positive statements that indicate progress along the health journey.

The nurse is caring for a patient who is immobile. The nurse frequently checks the patient for impaired skin integrity. What is the rationale for the nurse's action? a. Inadequate blood flow leads to decreased tissue ischemia. b. Patients with limited caloric intake develop thicker skin. c. Pressure reduces circulation to affected tissue. d. Verbalization of skin care needs is decreased.

ANS: C Body parts exposed to pressure have reduced circulation to affected tissue. Patients with limited caloric and protein intake develop thinner, less elastic skin with loss of subcutaneous tissue. Inadequate blood flow causes ischemia and breakdown. Verbalization is affected when altered cognition occurs from dementia, psychological disorders, or temporary delirium, not from immobility.

The nurse is cleansing a wound site. As the nurse administers the procedure, which intervention should be included? a. Allow the solution to flow from the most contaminated to the least contaminated. b. Scrub vigorously when applying noncytotoxic solution to the skin. c. Cleanse in a direction from the least contaminated area. d. Utilize clean gauze and clean gloves to cleanse a site.

ANS: C Cleanse in a direction from the least contaminated area, such as from the wound or incision, to the surrounding skin. While cleansing surgical or traumatic wounds by applying noncytotoxic solution with sterile gauze or by irrigations is correct, vigorous scrubbing is inappropriate and can cause damage to the skin. Use gentle friction when applying solutions to the skin, and allow irrigation to flow from the least to the most contaminated area.

A nurse is providing perineal care to a female patient. Which washing technique will the nurse use? a. Back to front b. In a circular motion c. From pubic area to rectum d. Upward from rectum to pubic area

ANS: C Cleansing from pubic area to rectum (front to back) reduces the transfer of microorganisms to the urinary meatus and decreases the risk of urinary tract infection. Cleansing from rectum to pubic area or back to front increases the risk of urinary tract infection. Circular motions are used in male perineal care.

An older adult's perineal skin is dry and thin with mild excoriation. When providing hygiene care after episodes of diarrhea, what should the nurse do? a. Thoroughly scrub the skin with a washcloth and hypoallergenic soap. b. Tape an occlusive moisture barrier pad to the patient's skin. c. Apply a skin protective ointment after perineal care. d. Massage the skin with light kneading pressure.

ANS: C Cleansing with a no-rinse cleanser and application of a barrier ointment should be done after each episode of diarrhea. Tape and occlusive dressings can damage skin. Excessive pressure and massage are inappropriate and may cause skin breakdown.

Vital signs for a patient reveal a high blood pressure of 187/100. Orders state to notify the health care provider for diastolic blood pressure greater than 90. What is the nurse's first action? a. Follow the clinical protocol for a stroke. b. Review the most recent lab results for the patient's potassium level. c. Assess the patient for other symptoms or problems, and then notify the health care provider. d. Administer an antihypertensive medication from the stock supply, and then notify the health care provider.

ANS: C Communication to other health care professionals must be timely, accurate, and relevant to a patient's clinical situation. The best answer is to reassess the patient for other symptoms or problems, and then notify the health care provider according to the orders. Reviewing the potassium level does not address the problem of high blood pressure. The nurse does not follow the protocol since the order says to notify the health care provider. The orders read to notify the health care provider, not administer medications.

A nurse is assessing the skin of an immobilized patient. What will the nurse do? a. Assess the skin every 4 hours. b. Limit the amount of fluid intake. c. Use a standardized tool such as the Braden Scale. d. Have special times for inspection so as to not interrupt routine care.

ANS: C Consistently use a standardized tool, such as the Braden Scale. This identifies patients with a high risk for impaired skin integrity. Skin assessment can be as often as every hour. Limiting fluids can lead to dehydration, increasing skin breakdown. Observe the skin often during routine care.

The nurse is caring for a patient with a pressure ulcer on the left hip. The ulcer is black. Which next step will the nurse anticipate? a. Monitor the wound. b. Document the wound. c. Debride the wound. d. Manage drainage from wound.

ANS: C Debridement is the removal of nonviable necrotic (black) tissue. Removal of necrotic tissue is necessary to rid the ulcer of a source of infection, to enable visualization of the wound bed, and to provide a clean base for healing. A wound will not move through the phases of healing if the wound is infected. Documentation occurs after completion of skill. When treating a pressure ulcer, it is important to monitor and reassess the wound at least every 8 hours. Management of drainage will help keep the wound clean, but that is not the next step.

A patient had an ileostomy surgically placed 2 days ago. Which diet will the nurse recommend to the patient to ease the transition of the new ostomy? a. Eggs over easy, whole wheat toast, and orange juice with pulp b. Chicken fried rice with fresh pineapple and iced tea c. Turkey meatloaf with white rice and apple juice d. Fish sticks with sweet corn and soda

ANS: C During the first few days after ostomy placement, the patient should consume easy-to-digest soft foods such as poultry, rice, and noodles. Fried foods can irritate digestion. Foods high in fiber will be useful later in the recovery process but can cause food blockage if the GI tract is not accustomed to digesting with an ileostomy. Foods with indigestible fiber such as sweet corn, popcorn, raw mushrooms, fresh pineapple, and Chinese cabbage could cause this problem.

A guaiac test is ordered for a patient. Which type of blood is the nurse checking for in this patient's stool? a. Bright red blood b. Dark black blood c. Microscopic d. Mucoid

ANS: C Fecal occult blood tests are used to test for blood that may be present in stool but cannot be seen by the naked eye (microscopic). This is usually indicative of a gastrointestinal bleed. All other options are incorrect. Detecting bright red blood, dark black blood, and blood that contains mucus (mucoid) is not the purpose of a guaiac test.

The nurse is caring for a patient who is experiencing a full-thickness repair. Which type of tissue will the nurse expect to observe when the wound is healing? a. Eschar b. Slough c. Granulation d. Purulent drainage

ANS: C Granulation tissue is red, moist tissue composed of new blood vessels, the presence of which indicates progression toward healing. Soft yellow or white tissue is characteristic of slough—a substance that needs to be removed for the wound to heal. Black or brown necrotic tissue is called eschar, which also needs to be removed for a wound to heal. Purulent drainage is indicative of an infection and will need to be resolved for the wound to heal.

The nurse is caring for a patient with a surgical incision that eviscerates. Which actions will the nurse take? (Select all that apply.) a. Place moist sterile gauze over the site. b. Gently place the organs back. c. Contact the surgical team. d. Offer a glass of water. e. Monitor for shock.

ANS: A, C, E The presence of an evisceration (protrusion of visceral organs through a wound opening) is a surgical emergency. Immediately place damp sterile gauze over the site, contact the surgical team, do not allow the patient anything by mouth (NPO), observe for signs and symptoms of shock, and prepare the patient for emergency surgery.

The nurse is teaching the parents of a child who has head lice (pediculosis capitis). Which information will the nurse include in the teaching session? a. Treatment is use of regular shampoo. b. Products containing lindane are most effective. c. Head lice may spread to furniture and other people. d. Manual removal is not a realistic option as treatment.

ANS: C Head lice are difficult to remove and spread to furniture and other people if not treated. Caution against use of products containing lindane because the ingredient is toxic and is known to cause adverse reactions. Treatments use medicated shampoo for eliminating lice. Manual removal is the best option when treatment has failed.

Which goal is most appropriate for a patient who has had a total hip replacement? a. The patient will ambulate briskly on the treadmill by the time of discharge. b. The patient will walk 100 feet using a walker by the time of discharge. c. The nurse will assist the patient to ambulate in the hall 2 times a day. d. The patient will ambulate by the time of discharge.

ANS: B "The patient will walk 100 feet using a walker by the time of discharge" is individualized, realistic, and measurable. "Ambulating briskly on a treadmill" is not realistic for this patient. The option that focuses on the nurse, not the patient, is not a measurable goal; this is an intervention. "The patient will ambulate by the time of discharge" is not measurable because it does not specify the distance. Even though we can see that the patient will ambulate, this does not quantify how far.

The nurse is teaching a new nurse about protocols. Which information from the new nurse indicates a correct understanding of the teaching? a. Protocols are guidelines to follow that replace the nursing care plan. b. Protocols assist the clinician in making decisions and choosing interventions for specific health care problems or conditions. c. Protocols are policies designating each nurse's duty according to standards of care and a code of ethics. d. Protocols are prescriptive order forms that help individualize the plan of care.

ANS: B A clinical practice guideline or protocol is a systematically developed set of statements that helps nurses, physicians, and other health care providers make decisions about appropriate health care for specific clinical situations. This guideline establishes interventions for specific health care problems or conditions. The protocol does not replace the nursing care plan. Evidence-based guidelines from protocols can be incorporated into an individualized plan of care. A clinical guideline is not the same as a hospital policy. Standing orders contain orders for the care of a specific group of patients. A protocol is not a prescriptive order form like a standing order.

The nurse is caring for a patient who is immobile and needs to be turned every 2 hours. The patient has poor lower extremity circulation, and the nurse is concerned about irritation of the patient's toes. Which device will the nurse use? a. Hand rolls b. A foot cradle c. A trapeze bar d. A trochanter roll

ANS: B A foot cradle may be used in patients with poor peripheral circulation as a means of reducing pressure on the tips of a patient's toes. A trochanter roll prevents external rotation of the hips when the patient is in a supine position. Hand rolls maintain the thumb in slight adduction and in opposition to the fingers. The trapeze bar is a triangular device that hangs down from a securely fastened overhead bar that is attached to the bedframe. It allows the patient to pull with the upper extremities to raise the trunk off the bed, to assist in transfer from bed to wheelchair, or to perform upper arm exercises.

The nurse is teaching a patient how to sit with crutches. In which order will the nurse present the instructions starting with the first step? 1. Place both crutches in one hand. 2. Grasp arm of chair with free hand. 3. Completely lower self into chair. 4. Transfer weight to crutches and unaffected leg. a. 4, 1, 2, 3 b. 1, 4, 2, 3 c. 1, 2, 4, 3 d. 4, 2, 1, 3

ANS: B A patient is sitting in a chair with crutches. Both crutches are held in one hand. The patient then transfers weight to the crutches and the unaffected leg. Next, the patient grasps the arm of the chair with the free hand and begins to lower self into chair. Finally, the patient completely lowers self into chair.

The female nurse is caring for a male patient who is uncircumcised but not ambulatory and has full function of all extremities. The nurse is providing the patient with a partial bed bath. How should perineal care be performed for this patient? a. Should be postponed because it may cause embarrassment b. Should be unnecessary because the patient is uncircumcised c. Should be done by the patient d. Should be done by the nurse

ANS: C If a patient is able to perform perineal self-care, encourage this independence. Patients most in need of perineal care are those at greatest risk for acquiring an infection such as uncircumcised males; perineal care is necessary. Embarrassment should not cause the nurse to overlook the patient's hygiene needs. The nurse should provide this care only if the patient is unable to do so.

The nurse is caring for a patient in the burn unit. Which type of wound healing will the nurse consider when planning care for this patient? a. Partial-thickness repair b. Secondary intention c. Tertiary intention d. Primary intention

ANS: B A wound involving loss of tissue such as a burn or a pressure ulcer or laceration heals by secondary intention. The wound is left open until it becomes filled with scar tissue. It takes longer for a wound to heal by secondary intention; thus the chance of infection is greater. A clean surgical incision is an example of a wound with little loss of tissue that heals by primary intention. The skin edges are approximated or closed, and the risk for infection is low. Partial-thickness repair is done on partial-thickness wounds that are shallow, involving loss of the epidermis and maybe partial loss of the dermis. These wounds heal by regeneration because the epidermis regenerates. Tertiary intention is seen when a wound is left open for several days, and then the wound edges are approximated. Wound closure is delayed until the risk of infection is resolved.

The patient is brought to the emergency department with possible injury to the left shoulder. Which area will the nurse assess to best determine joint mobility? a. The patient's gait b. The patient's range of motion c. The patient's ethnic influences d. The patient's fine-motor coordination

ANS: B Assessing range of motion is one assessment technique used to determine the degree of joint mobility and injury to a joint. Gait is the manner or style of walking. It has little bearing on the shoulder damage. Assessing fine-motor coordination would be beneficial in helping to assess the patient's ability to perform tasks such as feeding and dressing but would not help in evaluating the shoulder. Ethnic influences would not have a direct bearing on the amount of mobility in the joint.

Which instruction will the nurse provide to the nursing assistive personnel when providing foot care for a patient with diabetes? a. Do not place slippers on the patient's feet. b. Trim the patient's toenails daily. c. Report sores on the patient's toes. d. Check the brachial artery.

ANS: C Report any changes that may indicate inflammation or injury to tissue. Do not allow the diabetic patient to go barefoot; injury can lead to amputations. Clipping toenails is not allowed. Patients with peripheral vascular disease or diabetes mellitus often require nail care from a specialist to reduce the risk of infection. When assessing the patient's feet, the nurse palpates the dorsalis pedis of the foot, not the brachial artery.

The nurse is caring for a patient who is reporting severe foot pain due to corns. The patient has been using oval corn pads to self-treat the corns, but they seem to be getting worse. Which information will the nurse share with the patient? a. Corn pads are an adequate treatment and should be continued. b. The patient should avoid soaking the feet before using a pumice stone. c. Depending on severity, surgery may be needed to remove the corns. d. Tighter shoes would help to compress the corns and make them smaller.

ANS: C Surgical removal is necessary, depending on severity of pain and the size of the corn. Oval corn pads should be avoided because they increase pressure on the toes and reduce circulation. Warm water soaks soften corns before gentle rubbing with a callus file or pumice stone. Wider and softer shoes, especially shoes with a wider toe box, are helpful.

The nurse gives instructions to a nursing assistive personnel (NAP) regarding exercise for a patient. Which action by the NAP indicates a correct understanding of the directions? a. Determines the patient's ability to exercise b. Teaches the patient how to do the exercises c. Reports the patient got dizzy after exercising d. Advises the patient to work through the pain

ANS: C The NAP notifies the nurse if a patient reports increased fatigue, dizziness, or light-headedness when obtaining preexercise and/or postexercise vital signs. The nurse first must assess the patient's ability and tolerance to exercise. The nurse also teaches patients and their families how to implement exercise programs. The NAP can prepare patients for exercise (e.g., putting on shoes and clothing, providing hygiene needs, and obtaining preexercise and postexercise vital signs). The NAP can help the patient exercise.

The nurse is caring for a patient who is immobile. The nurse wants to decrease the formation of pressure ulcers. Which action will the nurse take first? a. Offer favorite fluids. b. Turn the patient every 2 hours. c. Determine the patient's risk factors. d. Encourage increased quantities of carbohydrates and fats.

ANS: C The first step in prevention is to assess the patient's risk factors for pressure ulcer development. When a patient is immobile, the major risk to the skin is the formation of pressure ulcers. Nursing interventions focus on prevention. Offering favorite fluids, turning, and increasing carbohydrates and fats are not the first steps. Determining risk factors is first so interventions can be implemented to reduce or eliminate those risk factors.

A nurse is making initial rounds on patients. Which intervention for a patient with poor wound healing should the nurse perform first? a. Reinforce the wound dressing as needed with 4 × 4 gauze. b. Perform the ordered dressing change twice daily. c. Observe wound appearance and edges. d. Document wound characteristics.

ANS: C The most appropriate initial intervention is to assess the wound (observe wound appearance and edges). The nurse must assess the wound first before the findings can be documented, reinforcement of the dressing, and the actual skill of dressing changes.

The patient is admitted to a skilled care unit for rehabilitation after the surgical procedure of fixation of a fractured left hip. The patient's nursing diagnosis is Impaired physical mobility related to musculoskeletal impairment from surgery and pain with movement. The patient is able to use a walker but needs assistance ambulating and transferring from the bed to the chair. Which nursing intervention is most appropriate for this patient? a. Obtain assistance and physically transfer the patient to the chair. b. Assist with ambulation and measure how far the patient walks. c. Give pain medication after ambulation so the patient will have a clear mind. d. Bring the patient to the cafeteria for group instruction on ambulation.

ANS: B Assist with walking and measure how far the patient walks to quantify progress. The nurse should allow the patient to do as much for self as possible. Therefore, the nurse should observe the patient transferring from the bed to the chair using the walker and should provide assistance as needed. The patient should be encouraged to use adequate pain medication to decrease the effects of pain and to increase mobility. The patient should be instructed on safe transfer and ambulation techniques in an environment with few distractions, not in the cafeteria.

The nurse is assessing the patient for respiratory complications of immobility. Which action will the nurse take when assessing the respiratory system? a. Inspect chest wall movements primarily during the expiratory cycle. b. Auscultate the entire lung region to assess lung sounds. c. Focus auscultation on the upper lung fields. d. Assess the patient at least every 4 hours.

ANS: B Auscultate the entire lung region to identify diminished breath sounds, crackles, or wheezes. Perform a respiratory assessment at least every 2 hours for patients with restricted activity. Inspect chest wall movements during the full inspiratory-expiratory cycle. Focus auscultation on the dependent lung fields because pulmonary secretions tend to collect in these lower regions.

The nurse is evaluating the body alignment of a patient in the sitting position. Which observation by the nurse will indicate a normal finding? a. The edge of the seat is in contact with the popliteal space. b. Both feet are supported on the floor with ankles flexed. c. The body weight is directly on the buttocks only. d. The arms hang comfortably at the sides.

ANS: B Both feet are supported on the floor, and the ankles are comfortably flexed. Body weight is evenly distributed on the buttocks and thighs. A 1- to 2-inch space is maintained between the edge of the seat and the popliteal space on the posterior surface of the knee to ensure that no pressure is placed on the popliteal artery or nerve. The patient's forearms are supported on the armrest, in the lap, or on a table in front of the chair.

A nurse is providing AM care to patients. Which action will the nurse take? a. Soaks feet of patient with peripheral vascular disease b. Applies CHG solution to wash perineum of patient with a stroke c. Cleanses eye from outer canthus to inner canthus of patient with diabetes d. Uses long, firm stroke to wash legs of patient with blood-clotting disorder

ANS: B CHG is safe to use on the perineum and external mucosa. If patient has diabetes or peripheral vascular disease with impaired circulation and/or sensation, do not soak feet. Maceration of skin may predispose to infection. Do not use long, firm strokes to wash the lower extremities of patients with history of deep vein thrombosis or blood-clotting disorders. Use short, light strokes instead. Eye should be cleansed from the inner to outer canthus on all patients.

A nurse is caring for a patient who has had diarrhea for the past week. Which additional assessment finding will the nurse expect? a. Distended abdomen b. Decreased skin turgor c. Increased energy levels d. Elevated blood pressure

ANS: B Chronic diarrhea can result in dehydration. Patients with chronic diarrhea are dehydrated with decreased skin turgor and blood pressure. Diarrhea also causes loss of electrolytes, nutrients, and fluid, which decreases energy levels. A distended abdomen could indicate constipation.

The nurse is caring for a patient with a healing Stage III pressure ulcer. The wound is clean and granulating. Which health care provider's order will the nurse question? a. Use a low-air-loss therapy unit. b. Irrigate with Dakin's solution. c. Apply a hydrogel dressing. d. Consult a dietitian.

ANS: B Clean pressure ulcers with noncytotoxic cleansers such as normal saline, which will not kill fibroblasts and healing tissue. Cytotoxic cleansers such as Dakin's solution, acetic acid, povidone-iodine, and hydrogen peroxide can hinder the healing process and should not be utilized on clean granulating wounds. Consulting a dietitian for the nutritional needs of the patient, utilizing a low-air-loss therapy unit to decrease pressure, and applying hydrogel dressings to provide a moist environment for healing are all orders that would be appropriate.

A nurse is caring for a patient with osteoporosis and lactose intolerance. What will the nurse do? a. Encourage dairy products. b. Monitor intake of vitamin D. c. Increase intake of caffeinated drinks. d. Try to do as much as possible for the patient.

ANS: B Encourage patients at risk to be screened for osteoporosis and assess their diets for calcium and vitamin D intake. Patients who have lactose intolerance need dietary teaching about alternative sources of calcium. Caffeine should be decreased. The goal of the patient with osteoporosis is to maintain independence with ADLs. Assistive ambulatory devices, adaptive clothing, and safety bars help the patient maintain independence.

A nurse is assessing a patient's skin. Which patient is most at risk for impaired skin integrity? a. A patient who is afebrile b. A patient who is diaphoretic c. A patient with strong pedal pulses d. A patient with adequate skin turgor

ANS: B Excessive moisture (diaphoretic) on the surface of the skin serves as a medium for bacterial growth and causes irritation, softens epidermal cells, and leads to skin maceration. A patient who is afebrile is not a high risk; however, a patient who is febrile (fever) is prone to skin breakdown. A patient with strong pedal pulses is not a high risk; however, a patient with vascular insufficiency is. A patient with adequate skin turgor is not a high risk; however, a patient with poor skin turgor is.

A patient's hygiene schedule of bathing and brushing teeth is largely influenced by family customs. For which age group is the nurse most likely providing care? a. Adolescent b. Preschooler c. Older adult d. Adult

ANS: B Family customs play a major role during childhood in determining hygiene practices such as the frequency of bathing, the time of day bathing is performed, and even whether certain hygiene practices such as brushing of the teeth or flossing are performed. As children enter adolescence, peer groups and media often influence hygiene practices. During the adult years involvement with friends and work groups shape the expectations that people have about personal appearance. Some older adults' hygiene practices change because of changes in living conditions and available resources.

The nurse is caring for a patient who has a Stage IV pressure ulcer with grafted surgical sites. Which specialty bed will the nurse use for this patient? a. Low-air-loss b. Air-fluidized c. Lateral rotation d. Standard mattress

ANS: B For a patient with newly flapped or grafted surgical sites, the air-fluidized bed will be the best choice; this uses air and fluid support to provide pressure redistribution via a fluid-like medium created by forcing air through beads as characterized by immersion and envelopment. A low-air-loss bed is utilized for prevention or treatment of skin breakdown by preventing buildup of moisture and skin breakdown through the use of airflow. A standard mattress is utilized for an individual who does not have actual or potential altered or impaired skin integrity. Lateral rotation is used for treatment and prevention of pulmonary, venous stasis and urinary complications associated with mobility.

A nurse is evaluating care of an immobilized patient. Which action will the nurse take? a. Focus on whether the interdisciplinary team is satisfied with the care. b. Compare the patient's actual outcomes with the outcomes in the care plan. c. Involve primarily the patient's family and health care team to determine goal achievement. d. Use objective data solely in determining whether interventions have been successful.

ANS: B From your perspective as the nurse, you are to evaluate outcomes and response to nursing care and compare the patient's actual outcomes with the outcomes selected during planning. Ask if the patient's expectations (subjective data) of care are being met, and use objective data to determine the success of interventions. Just as it was important to include the patient during the assessment and planning phase of the care plan, it is essential to have the patient's evaluation of the plan of care, not just the patient's family and health care team.

A new nurse is working in a unit that uses interdisciplinary collaboration. Which action will the nurse take? a. Act as a leader of the health care team. b. Develop good communication skills. c. Work solely with nurses. d. Avoid conflict.

ANS: B Good communication between other health care providers builds trust and is related to the acceptance of your role in the health care team. As a beginning nurse, you will not be considered a leader of the health care team, but your input as an interdisciplinary team member is critical. Interdisciplinary involves other health care providers, not just nurses. Organizational culture includes leadership, communication processes, shared beliefs about the quality of clinical guidelines, and conflict resolution.

The nurse is teaching a patient how to use a cane. Which information will the nurse include in the teaching session? a. Place the cane at the top of the hip bone. b. Place the cane on the stronger side of the body. c. Place the cane in front of the body and then move the good leg. d. Place the cane 10 to 15 inches in front of the body when walking.

ANS: B Have the patient keep the cane on the stronger side of the body. A person's cane length is equal to the distance between the greater trochanter and the floor. The cane should be moved first and then the weaker leg. For maximum support when walking, the patient places the cane forward 15 to 25 cm (6 to 10 inches), keeping body weight on both legs. The weaker leg is then moved forward to the cane, so body weight is divided between the cane and the stronger leg.

The patient is immobilized after undergoing hip replacement surgery. Which finding will alert the nurse to monitor for hemorrhage in this patient? a. Thick, tenacious pulmonary secretions b. Low-molecular-weight heparin doses c. SCDs wrapped around the legs d. Elastic stockings (TED hose)

ANS: B Heparin and low-molecular-weight heparin are the most widely used drugs in the prophylaxis of deep vein thrombosis. Because bleeding is a potential side effect of these medications, continually assess the patient for signs of bleeding. Pulmonary secretions that become thick and tenacious are difficult to remove and are a sign of inadequate hydration or developing pneumonia but not of bleeding. SCDs consist of sleeves or stockings made of fabric or plastic that are wrapped around the leg and are secured with Velcro. They decrease venous stasis by increasing venous return through the deep veins of the legs. They do not usually cause bleeding. Elastic stockings also aid in maintaining external pressure on the muscles of the lower extremities and in promoting venous return. They do not usually cause bleeding.

Before administering a cleansing enema to an 80-year-old patient, the patient says "I don't think I will be able to hold the enema." Which is the next priority nursing action? a. Rolling the patient into right-lying Sims' position b. Positioning the patient in the dorsal recumbent position on a bedpan c. Inserting a rectal plug to contain the enema solution after administering d. Assisting the patient to the bedside commode and administering the enema

ANS: B If you suspect the patient of having poor sphincter control, position on bedpan in a comfortable dorsal recumbent position. Patients with poor sphincter control are unable to retain all of the enema solution. Administering an enema with the patient sitting on the toilet is unsafe because it is impossible to safely guide the tubing into the rectum, and it will be difficult for the patient to retain the fluid as he or she is in the position used for emptying the bowel. Rolling the patient into right-lying Sims' position will not help the patient retain the enema. Use of a rectal plug to contain the solution is inappropriate and unsafe.

A nurse is assessing pressure points in a patient placed in the Sims' position. Which areas will the nurse observe? a. Chin, elbow, hips b. Ileum, clavicle, knees c. Shoulder, anterior iliac spine, ankles d. Occipital region of the head, coccyx, heels

ANS: B In the Sims' position pressure points include the ileum, humerus, clavicle, knees, and ankles. The lateral position pressure points include the ear, shoulder, anterior iliac spine, and ankles. The prone position pressure points include the chin, elbows, female breasts, hips, knees, and toes. Supine position pressure points include the occipital region of the head, vertebrae, coccyx, elbows, and heels.

A patient visiting with family members in the waiting area tells the nurse "I don't feel good, especially in the stomach." What should the nurse do? a. Request that the family leave, so the patient can rest. b. Ask the patient to return to the room, so the nurse can inspect the abdomen. c. Ask the patient when the last bowel movement was and to lie down on the sofa. d. Tell the patient that the dinner tray will be ready in 15 minutes and that may help the stomach feel better.

ANS: B In this case, the environment needs to be conducive to completing a thorough assessment. A patient's care environment needs to be safe and conducive to implementing therapies. When you need to expose a patient's body parts, do so privately by closing room doors or curtains because the patient will then be more relaxed; the patient needs to return to the room for an abdominal assessment for privacy and comfort. The family can remain in the waiting area while the nurse assists the patient back to the room. Beginning the assessment in the waiting area (lie down on the sofa) in the presence of family and other visitors does not promote privacy and patient comfort. Telling the patient that the dinner tray is almost ready is making an assumption that the abdominal discomfort is due to not eating. The nurse needs to perform an assessment first.

A patient with diabetes mellitus is starting an exercise program. Which types of exercises will the nurse suggest? a. Low intensity b. Low to moderate intensity c. Moderate to high intensity d. High intensity

ANS: B Instruct patients diagnosed with diabetes mellitus to perform low- to moderate-intensity exercises, carry a concentrated form of carbohydrates (sugar packets or hard candy), and wear a medical alert bracelet. Low intensity is not beneficial. Moderate to high and high intensity are not recommended for a beginner exercise program.

The nurse is developing a plan of care for a patient diagnosed with activity intolerance. Which strategy will the nurse use to provide the best chance of maintaining patient compliance? a. Performing 20 minutes of aerobic exercise 7 days a week with 10-minute warm-up and cool-down periods b. Instructing the patient to use an exercise log to record day, time, duration, and responses to exercise activity c. Stressing the harm of not exercising by getting the patient to take responsibility for current health status d. Arranging for the patient to join a gym that takes self-pay rather than insurance

ANS: B Keeping a log may increase adherence to an exercise prescription. Recommended frequency of aerobic exercise is 3 to 5 times per week or every other day for approximately 30 minutes. Focusing on the harm of not exercising is usually counterproductive. Instead, the nurse should instruct the patient about the physiological benefits of a regular exercise program. Developing a plan of exercise that the patient may perform at home may improve compliance.

An adolescent tells the nurse that a health professional said the fibrous tissue that connects bone and cartilage was strained in a sporting accident. On which structure will the nurse focus an assessment? a. Tendon b. Ligament c. Synergistic muscle d. Antagonistic muscle

ANS: B Ligaments are white, shiny, and flexible bands of fibrous tissue that bind joints and connect bones and cartilage. Tendons are strong, flexible, and inelastic as they serve to connect muscle to bone. Muscles attach bone to bone. Synergistic muscles contract to accomplish the same movement. Antagonistic muscles cause movement at the joint.

The nurse is assessing an immobile patient for deep vein thromboses (DVTs). Which action will the nurse take? a. Remove elastic stockings every 4 hours. b. Measure the calf circumference of both legs. c. Lightly rub the lower leg for redness and tenderness. d. Dorsiflex the foot while assessing for patient discomfort.

ANS: B Measure bilateral calf circumference and record it daily as an assessment for DVT. Unilateral increases in calf circumference are an early indication of thrombosis. Homan's sign, or calf pain on dorsiflexion of the foot, is no longer a reliable indicator in assessing for DVT, and it is present in other conditions. Remove the patient's elastic stockings and/or sequential compression devices (SCDs) every 8 hours, and observe the calves for redness, warmth, and tenderness. Instruct the family, patient, and all health care personnel not to massage the area because of the danger of dislodging the thrombus.

A nurse is preparing to provide hygiene care. Which principle should the nurse consider when planning hygiene care? a. Hygiene care is always routine and expected. b. No two individuals perform hygiene in the same manner. c. It is important to standardize a patient's hygienic practices. d. During hygiene care do not take the time to learn about patient needs.

ANS: B No two individuals perform hygiene in the same manner; it is important to individualize the patient's care based on knowing about the patient's unique hygiene practices and preferences. Hygiene care is never routine; this care requires intimate contact with the patient and communication skills to promote the therapeutic relationship. In addition, during hygiene, the nurse should take time to learn about the patient's health promotion practices and needs, emotional needs, and health care education needs.

A nurse is caring for a patient with a wound. Which assessment data will be most important for the nurse to gather with regard to wound healing? a. Muscular strength assessment b. Pulse oximetry assessment c. Sensation assessment d. Sleep assessment

ANS: B Oxygen fuels the cellular functions essential to the healing process; the ability to perfuse tissues with adequate amounts of oxygenated blood is critical in wound healing. Pulse oximetry measures the oxygen saturation of blood. Assessment of muscular strength and sensation, although useful for fitness and mobility testing, does not provide any data with regard to wound healing. Sleep, although important for rest and for integration of learning and restoration of cognitive function, does not provide any data with regard to wound healing.

The nurse is caring for a patient who has had a stroke causing total paralysis of the right side. To help maintain joint function and minimize the disability from contractures, passive ROM will be initiated. When should the nurse begin this therapy? a. After the acute phase of the disease has passed b. As soon as the ability to move is lost c. Once the patient enters the rehab unit d. When the patient requests it

ANS: B Passive ROM exercises should begin as soon as the patient's ability to move the extremity or joint is lost. The nurse should not wait for the acute phase to end. It may be some time before the patient enters the rehab unit or the patient requests it, and contractures could form by then.

A patient is receiving opioids for pain. Which bowel assessment is a priority? a. Clostridium difficile b. Constipation c. Hemorrhoids d. Diarrhea

ANS: B Patients receiving opiates for pain after surgery often require a stool softener or laxative to prevent constipation. C. difficile occurs from antibiotics, not opioids. Hemorrhoids are caused by conditions other than opioids. Diarrhea does not occur as frequently as constipation.

A nurse is pouching an ostomy on a patient with an ileostomy. Which action by the nurse is most appropriate? a. Changing the skin barrier portion of the ostomy pouch daily b. Emptying the pouch if it is more than one-third to one-half full c. Thoroughly cleansing the skin around the stoma with soap and water to remove excess stool and adhesive d. Measuring the correct size for the barrier device while leaving a 1/2-inch space around the stoma

ANS: B Pouches must be emptied when they are one-third to one-half full because the weight of the pouch may disrupt the seal of the adhesive on the skin. The barrier device should be changed every 3 to 7 days unless it is leaking or is no longer effective. Peristomal skin should be gently cleansed; vigorous rubbing can cause further irritation or skin breakdown. Avoid soap. It leaves a residue on skin, which may irritate the skin. The pouch opening should fit around the stoma and cover the peristomal skin to prevent contact with the effluent. Excess space, like 1/2 inch, allows fecal matter to have prolonged exposure to skin, resulting in skin breakdown.

The nurse is caring for a patient who was involved in an automobile accident 2 weeks ago. The patient sustained a head injury and is unconscious. Which priority element will the nurse consider when planning care to decrease the development of a decubitus ulcer? a. Resistance b. Pressure c. Weight d. Stress

ANS: B Pressure is the main element that causes pressure ulcers. Three pressure-related factors contribute to pressure ulcer development: pressure intensity, pressure duration, and tissue tolerance. When the intensity of the pressure exerted on the capillary exceeds 15 to 32 mm Hg, this occludes the vessel, causing ischemic injury to the tissues it normally feeds. High pressure over a short time and low pressure over a long time cause skin breakdown. Resistance, stress, and weight are not the priority causes of pressure ulcers.

The medical-surgical acute care patient has received a nursing diagnosis of Impaired skin integrity. Which health care team member will the nurse consult? a. Respiratory therapist b. Registered dietitian c. Case manager d. Chaplain

ANS: B Refer patients with pressure ulcers to the dietitian for early intervention for nutritional problems. Adequate calories, protein, vitamins, and minerals promote wound healing for the impaired skin integrity. The nurse is the coordinator of care, and collaborating with the dietitian would result in planning the best meals for the patient. The respiratory therapist can be consulted when a patient has issues with the respiratory system. Case management can be consulted when the patient has a discharge need. A chaplain can be consulted when the patient has a spiritual need.

A nurse reviews an immobilized patient's laboratory results and discovers hypercalcemia. Which condition will the nurse monitor for most closely in this patient? a. Hypostatic pneumonia b. Renal calculi c. Pressure ulcers d. Thrombus formation

ANS: B Renal calculi are calcium stones that lodge in the renal pelvis or pass through the ureters. Immobilized patients are at risk for calculi because they frequently have hypercalcemia. Hypercalcemia does not lead to hypostatic pneumonia, pressure ulcers, or thrombus formation. Immobility is one cause of hypostatic pneumonia, which is inflammation of the lung from stasis or pooling of secretions. A pressure ulcer is an impairment of the skin that results from prolonged ischemia (decreased blood supply) within tissues. A thrombus is an accumulation of platelets, fibrin, clotting factors, and cellular elements of the blood attached to the interior wall of a vein or artery, which sometimes occludes the lumen of the vessel.

The nurse is caring for a patient with a Stage IV pressure ulcer. Which type of healing will the nurse consider when planning care for this patient? a. Partial-thickness wound repair b. Full-thickness wound repair c. Primary intention d. Tertiary intention

ANS: B Stage IV pressure ulcers are full-thickness wounds that extend into the dermis and heal by scar formation because the deeper structures do not regenerate, hence the need for full-thickness repair. The full-thickness repair has four phases: hemostasis, inflammatory, proliferative, and maturation. A wound heals by primary intention when wounds such as surgical wounds have little tissue loss; the skin edges are approximated or closed, and the risk for infection is low. Partial-thickness repairs are done on partial-thickness wounds that are shallow, involving loss of the epidermis and maybe partial loss of the dermis. These wounds heal by regeneration because the epidermis regenerates. Tertiary intention is seen when a wound is left open for several days, and then the wound edges are approximated. Wound closure is delayed until risk of infection is resolved.

A nurse is assisting the patient to perform isometric exercises. Which action will the nurse take? a. Encourage wearing tight shoes. b. Set the pace for the exercise session. c. Stop the exercise if pain is experienced. d. Force muscles or joints to go just beyond resistance.

ANS: C Instruct the patient to stop the activity if pain, fatigue, or discomfort is experienced. Assess for pain, shortness of breath, or a change in vital signs; if present, stop the exercise. Let each patient move at his or her own pace. Assess for joint limitations, and do not force a muscle or a joint during exercise. Teach patient to wear comfortable shoes and clothing for exercise.

A nurse is checking orders. Which order should the nurse question? a. A normal saline enema to be repeated every 4 hours until stool is produced b. A hypertonic solution enema for a patient with fluid volume excess c. A Kayexalate enema for a patient with severe hypokalemia d. An oil retention enema for a patient with constipation

ANS: C Kayexalate binds to and helps excrete potassium, so it would be contraindicated in patients who are hypokalemic (have low potassium). Normal saline enemas can be repeated without risk of fluid or electrolyte imbalance. Hypertonic solutions are intended for patients who cannot handle large fluid volume and are contraindicated for dehydrated patients. Oil retention enemas lubricate the feces in the rectum and colon and are used for constipation.

A patient has developed a pressure ulcer. Which laboratory data will be important for the nurse to check? a. Vitamin E b. Potassium c. Albumin d. Sodium

ANS: C Normal wound healing requires proper nutrition. Serum proteins are biochemical indicators of malnutrition, and serum albumin is probably the most frequently measured of these parameters. The best measurement of nutritional status is prealbumin because it reflects not only what the patient has ingested but also what the body has absorbed, digested, and metabolized. Zinc and copper are the minerals important for wound healing, not potassium and sodium. Vitamins A and C are important for wound healing, not vitamin E.

The nurse inserts an intravenous (IV) catheter using the correct technique and following the recommended steps according to standards of care and hospital policy. Which type of implementation skill is the nurse using? a. Cognitive b. Interpersonal c. Psychomotor d. Judgmental

ANS: C Nursing practice includes cognitive, interpersonal, and psychomotor skills. Psychomotor skill requires the integration of cognitive and motor abilities. The nurse in this example displayed the psychomotor skill of inserting an intravenous catheter while following standards of care and integrating knowledge of anatomy and physiology. Cognitive involves the application of critical thinking and use of good judgment in making sound clinical decisions. Interpersonal skills involve developing trusting relationships with patients, conveying caring and compassion, and communicating clearly.

A nurse is assessing a patient with activity intolerance for possible orthostatic hypotension. Which finding will help confirm orthostatic hypotension? a. Blood pressure sitting 120/64; blood pressure 140/70 standing b. Blood pressure sitting 126/64; blood pressure 120/58 standing c. Blood pressure sitting 130/60; blood pressure 110/60 standing d. Blood pressure sitting 140/60; blood pressure 130/54 standing

ANS: C Orthostatic hypotension results in a drop of 20 mm Hg systolic or more in blood pressure when rising from sitting position (110/60). 120 to 140 means the blood pressure increased rather than dropped. 126 to 120 is only a six points' difference. 140 to 130 is only a 10 points' difference.

Which patient is most at risk for increased peristalsis? a. A 5-year-old child who ignores the urge to defecate owing to embarrassment b. A 21-year-old female with three final examinations on the same day c. A 40-year-old female with major depressive disorder d. An 80-year-old male in an assisted-living environment

ANS: B Stress can stimulate digestion and increase peristalsis, resulting in diarrhea; three finals on the same day is stressful. Ignoring the urge to defecate, depression, and age-related changes of the older adult (80-year-old man) are causes of constipation, which is from slowed peristalsis.

The nurse is caring for a patient in the emergency department with an injured shoulder. Which type of joint will the nurse assess? a. Fibrous b. Synovial c. Synergistic d. Cartilaginous

ANS: B Synovial joints, or true joints, such as the hinge type at the elbow, are freely movable and the most mobile, numerous, and anatomically complex body joints. Fibrous joints fit closely together and are fixed, permitting little, if any, movement such as the syndesmosis between the tibia and the fibula. Synergistic is a type of muscle, not joint. Cartilaginous joints have little movement but are elastic and use cartilage to unite separate bony surfaces such as the synchondrosis that attaches the ribs to the costal cartilage.

The nurse collects the following assessment data: right heel with reddened area that does not blanch. Which nursing diagnosis will the nurse assign to this patient? a. Imbalanced nutrition: less than body requirements b. Ineffective peripheral tissue perfusion c. Risk for infection d. Acute pain

ANS: B The area on the heel has experienced a decreased supply of blood and oxygen (tissue perfusion), which has resulted in tissue damage. The most appropriate nursing diagnosis with this information is Ineffective peripheral tissue perfusion. Risk for infection, Acute pain, and Imbalanced nutrition do not support the data in the question.

Which nursing intervention is most effective in promoting normal defecation for a patient who has muscle weakness in the legs? a. Administer a soapsuds enema every 2 hours. b. Use a mobility device to place the patient on a bedside commode. c. Give the patient a pillow to brace against the abdomen while bearing down. d. Elevate the head of the bed 20 degrees 60 minutes after breakfast while on bedpan.

ANS: B The best way to promote normal defecation is to assist the patient into a posture that is as normal as possible for defecation. Using a mobility device promotes nurse and patient safety. Elevating the head of the bed is appropriate but is not the most effective; closer to 30 to 45 degrees is the proper position for the patient on a bedpan, and the patient is not on bed rest so a bedside commode is the best choice. Giving the patient a pillow may reduce discomfort, but this is not the best way to promote defecation. A soapsuds enema is indicated for a patient who needs assistance to stimulate peristalsis. It promotes non-natural defecation.

Which patient will cause the nurse to select a nursing diagnosis of Impaired physical mobility for a care plan? a. A patient who is completely immobile b. A patient who is not completely immobile c. A patient at risk for single-system involvement d. A patient who is at risk for multisystem problems

ANS: B The diagnosis of Impaired physical mobility applies to the patient who has some limitation but is not completely immobile. The diagnosis of Risk for disuse syndrome applies to the patient who is immobile and at risk for multisystem problems because of inactivity. Beyond these diagnoses, the list of potential diagnoses is extensive because immobility affects multiple body systems.

The nurse is providing a complete bed bath to a patient using a commercial bath cleansing pack (bag bath). What should the nurse do? a. Rinse thoroughly. b. Allow the skin to air-dry. c. Do not use a bath towel. d. Dry the skin with a towel.

ANS: B The nurse should allow the skin to air-dry for 30 seconds. Drying the skin with a towel removes the emollient that is left behind after the water/cleanser solution evaporates. It is permissible to lightly cover the patient with a bath blanket or towel to prevent chilling. Do not rinse when using a bag bath.

A nurse is providing a bath. In which order will the nurse clean the body, beginning with the first area? 1. Face 2. Eyes 3. Perineum 4. Arm and chest 5. Hands and nails 6. Back and buttocks 7. Abdomen and legs a. 1, 2, 5, 4, 7, 6, 3 b. 2, 1, 4, 5, 7, 3, 6 c. 2, 1, 5, 4, 6, 7, 3 d. 1, 2, 4, 5, 3, 7, 6

ANS: B The sequence for giving a bath is as follows: eyes, face, both arms, chest, hands/nails, abdomen, both legs, perineal hygiene, back, and buttocks/anus.

A nurse is preparing to reposition a patient. Which task can the nurse delegate to the nursing assistive personnel? a. Determining the level of comfort b. Changing the patient's position c. Identifying immobility hazards d. Assessing circulation

ANS: B The skill of moving and positioning patients in bed can be delegated to nursing assistive personnel (NAP). The nurse is responsible for assessing the patient's level of comfort and for any hazards of immobility and assessing circulation.

The nurse is performing a moist-to-dry dressing. The nurse has prepared the supplies, solution, and removed the old dressing. In which order will the nurse implement the steps, starting with the first one? 1. Apply sterile gloves. 2. Cover and secure topper dressing. 3. Assess wound and surrounding skin. 4. Moisten gauze with prescribed solution. 5. Gently wring out excess solution and unfold. 6. Loosely pack until all wound surfaces are in contact with gauze. a. 4, 3, 1, 5, 6, 2 b. 1, 3, 4, 5, 6, 2 c. 4, 1, 3, 5, 6, 2 d. 1, 4, 3, 5, 6, 2

ANS: B The steps for a moist-to-dry dressing are as follows: (1) Apply sterile gloves; (2) assess appearance of surrounding skin; (3) moisten gauze with prescribed solution. (4) Gently wring out excess solution and unfold; apply gauze as single layer directly onto wound surface. (5) If wound is deep, gently pack dressing into wound base by hand until all wound surfaces are in contact with gauze; (6) cover with sterile dry gauze and secure topper dressing.

The patient has been diagnosed with diabetes. When admitted, the patient is unkempt and is in need of a bath and foot care. When questioned about hygiene habits, the nurse learns the patient takes a bath once a week and a sponge bath every other day. To provide ultimate care for this patient, which principle should the nurse keep in mind? a. Patients who appear unkempt place little importance on hygiene practices. b. Personal preferences determine hygiene practices and are unchangeable. c. The patient's illness may require teaching of new hygiene practices. d. All cultures value cleanliness with the same degree of importance.

ANS: C The nurse must assist the patient in developing new hygiene practices when indicated by an illness or condition. For example, the nurse will need to teach a patient with diabetes proper foot hygiene. Patients who appear unkempt often need further assessment regarding their ability to participate in daily hygiene. Patients with certain types of physical limitations or disabilities often lack the physical energy and dexterity to perform hygienic care. Culturally, maintaining cleanliness does not hold the same importance for some ethnic groups as it does for others.

The nurse is caring for a group of patients. Which patient will the nurse see first? a. A patient with chronic obstructive pulmonary disease doing stretching exercises b. A patient with diabetes mellitus carrying hard candy while doing exercises c. A patient with a heart attack doing isometric exercises d. A patient with hypertension doing Tai Chi exercises

ANS: C The nurse must see the myocardial infarction patient first to stop this type of exercise. It is important to understand the energy expenditure (increased respiratory rate and increased work on the heart) associated with isometric exercises because the exercises are sometimes contraindicated in certain patients' illnesses (e.g., myocardial infarction or chronic obstructive pulmonary disease). All the rest are appropriate. Stretching exercises are beneficial for patients with chronic obstructive pulmonary disease. Also instruct patients to perform low- to moderate-intensity exercises, carry a concentrated form of carbohydrates (sugar packets or hard candy), and wear a medical alert bracelet. The effect of a Tai Chi exercise program has demonstrated a significant reduction in systolic and diastolic blood pressures.

The nurse is caring for a patient who cannot bear weight but needs to be transferred from the bed to a chair. The nurse decides to use a transportable hydraulic lift. What will the nurse do? a. Place a horseshoe-shaped base on the opposite side from the chair. b. Remove straps before lowering the patient to the chair. c. Hook longer straps to the bottom of the sling. d. Attach short straps to the bottom of the sling.

ANS: C The nurse should attach the hooks on the strap to the holes in the sling. Short straps hook to top holes of the sling; longer straps hook to the bottom of the sling. The horseshoe-shaped base goes under the side of the bed on the side with the chair. Position the patient and lower slowly into the chair in accordance with manufacturer guidelines to safely guide the patient into the back of the chair as the seat descends; then remove the straps and the mechanical/hydraulic lift.

Which behavior indicates the nurse is using a team approach when caring for a patient who is experiencing alterations in mobility? a. Delegates assessment of lung sounds to nursing assistive personnel b. Becomes solely responsible for modifying activities of daily living c. Consults physical therapy for strengthening exercises in the extremities d. Involves respiratory therapy for altered breathing from severe anxiety levels

ANS: C The nurse should collaborate with other health care team members such as physical or occupational therapists when considering mobility needs. For example, physical therapists are a resource for planning ROM or strengthening exercises. Nurses often delegate some interventions to nursing assistive personnel, but assessment of lung sounds is the nurse's responsibility. Nursing assistive personnel may turn and position patients, apply elastic stockings, help patients use the incentive spirometer, etc. Occupational therapists are a resource for planning activities of daily living that patients need to modify or relearn. A mental health advanced practice nurse or psychologist should be used for severe anxiety.

The nurse is caring for a group of patients. Which patient will the nurse see first? a. A patient with a Stage IV pressure ulcer b. A patient with a Braden Scale score of 18 c. A patient with appendicitis using a heating pad d. A patient with an incision that is approximated

ANS: C The nurse should see the patient with an appendicitis first. Warm applications are contraindicated when the patient has an acute, localized inflammation such as appendicitis because the heat could cause the appendix to rupture. Although a Stage IV pressure ulcer is deep, it is not as critical as the appendicitis patient. The total Braden score ranges from 6 to 23; a lower total score indicates a higher risk for pressure ulcer development. A score of 18 can be assessed later. A healing incision is approximated (closed); this is a normal finding and does not need to be seen first.

The nurse is caring for patients with ostomies. In which ostomy location will the nurse expect very liquid stool to be present? a. Sigmoid b. Transverse c. Ascending d. Descending

ANS: C The path of digestion goes from the ascending, across the transverse, to the descending and finally passing into the sigmoid; therefore, the least formed stool (very liquid) would be in the ascending.

The nurse is caring for a patient with diabetes. Which task will the nurse assign to the nursing assistive personnel? a. Providing nail care b. Teaching foot care c. Making an occupied bed d. Determining aspiration risk

ANS: C The skill of making an occupied bed can be delegated to nursing assistive personnel. Nail care, teaching foot care, and assessing aspiration risk of a patient with diabetes must be performed by the RN; these skills cannot be delegated.

Which action will the nurse take to reduce the risk of excoriation to the mucosal lining of the patient's nose from a nasogastric tube? a. Instill Xylocaine into the nares once a shift. b. Tape tube securely with light pressure on nare. c. Lubricate the nares with water-soluble lubricant. d. Apply a small ice bag to the nose for 5 minutes every 4 hours.

ANS: C The tube constantly irritates the nasal mucosa, increasing the risk of excoriation. Frequent lubrication with a water-soluble lubricant decreases the likelihood of excoriation and is less toxic than oil-based if aspirated. Xylocaine is used to treat sore throat, not nasal mucosal excoriation. While the tape should be secure, pressure will increase excoriation. Ice is not applied to the nose.

The wound care nurse visits a patient in the long-term care unit. The nurse is monitoring a patient with a Stage III pressure ulcer. The wound seems to be healing, and healthy tissue is observed. How should the nurse document this ulcer in the patient's medical record? a. Stage I pressure ulcer b. Healing Stage II pressure ulcer c. Healing Stage III pressure ulcer d. Stage III pressure ulcer

ANS: C When a pressure ulcer has been staged and is beginning to heal, the ulcer keeps the same stage and is labeled with the words "healing stage" or healing Stage III pressure ulcer. Once an ulcer has been staged, the stage endures even as the ulcer heals. This ulcer was labeled a Stage III, and it cannot return to a previous stage such as Stage I or II. This ulcer is healing, so it is no longer labeled a Stage III.

The patient reports to the nurse about a perceived decrease in hearing. When the nurse examines the patient's ear, a large amount of cerumen buildup at the entrance to the ear canal is observed. Which action will the nurse take next? a. Teach the patient how to use cotton-tipped applicators. b. Tell the patient to use a bobby pin to extract earwax. c. Apply gentle, downward retraction of the ear canal. d. Instill hot water into the ear canal to melt the wax.

ANS: C When cerumen is visible, gentle, downward retraction at the entrance to the ear canal causes the wax to loosen and slip out. Instruct the patient never to use sharp objects such as bobby pins or paper clips to remove earwax. Use of such objects can traumatize the ear canal and ruptures the tympanic membrane. Avoid the use of cotton-tipped applicators as well because they cause earwax to become impacted within the canal. Instilling cold or hot water causes nausea or vomiting.

The nurse is evaluating care of a patient for crutches. Which finding indicates a successful outcome? a. The top of the crutch is three to four finger widths from the armpit. b. The elbows are slightly flexed at 30 to 35 degrees when the patient is standing. c. The tip of the crutch is 4 to 6 inches anterior to the front of the patient's shoes. d. The position of the handgrips allows the axilla to support the patient's body weight.

ANS: C When crutches are fitted, the tip of the crutch is 4 to 6 inches anterior to the front of the patient's shoes, and the length of the crutch is two to three finger widths from the axilla. Position the handgrips so the axillae are not supporting the patient's body weight. Pressure on the axillae increases risk to underlying nerves, which sometimes results in partial paralysis of the arm. Determine correct position of the handgrips with the patient upright, supporting weight by the handgrips with the elbows slightly flexed at 20 to 25 degrees.

The nurse is completing a skin risk assessment using the Braden Scale. The patient has slight sensory impairment, has skin that is rarely moist, walks occasionally, and has slightly limited mobility, along with excellent intake of meals and no apparent problem with friction and shear. Which score will the nurse document for this patient? a. 15 b. 17 c. 20 d. 23

ANS: C With use of the Braden Scale, the total score is a 20. The patient receives 3 for slight sensory perception impairment, 4 for skin being rarely moist, 3 for walks occasionally, 3 for slightly limited mobility, 4 for intake of meals, and 4 for no problem with friction and shear.

The patient reports being tired and weak and lacks energy. Upon assessment, the nurse finds that patient has gained weight, and blood pressure and pulse are elevated after climbing stairs. Which nursing diagnosis will the nurse add to the care plan? a. Fatigue b. Ineffective coping c. Activity intolerance d. Decreased cardiac output

ANS: C You consider nursing diagnoses of Activity intolerance or Fatigue in a patient who reports being tired and weak. Further review of assessed defining characteristics (e.g., abnormal heart rate and verbal report of weakness and the assessment findings occurring during the activity of climbing the stairs) leads to the definitive diagnosis (Activity intolerance). There is no data to support ineffective coping or decreased cardiac output.

A nurse is implementing interventions for a group of patients. Which actions are nursing interventions?(Select all that apply.) a. Order chest x-ray for suspected arm fracture. b. Prescribe antibiotics for a wound infection. c. Reposition a patient who is on bed rest. d. Teach a patient preoperative exercises. e. Transfer a patient to another hospital unit.

ANS: C, D, E A nursing intervention is any treatment based on clinical judgment and knowledge that a nurse performs to enhance patient outcomes. Repositioning, teaching, and transferring a patient are examples of nursing interventions. Ordering a chest x-ray and prescribing antibiotics are examples of medical interventions performed by a health care provider.

The patient must stay in bed for a bed change. Which actions will the nurse implement? (Select all that apply.) a. Apply sterile gloves. b. Keep soiled linen close to uniform. c. Advise patient will feel a lump when rolling over. d. Turn clean pillowcase inside out over the hand holding it. e. Make a modified mitered corner with sheet, blanket, and spread.

ANS: C, D, E When making an occupied bed, advise patients they will feel a lump when turning, turn clean pillowcase inside out, and make a modified mitered corner. Clean gloves are used. Keep soiled linen away from uniform.

The nurse is caring for a patient who has experienced a laparoscopic appendectomy. For which type of healing will the nurse focus the care plan? a. Partial-thickness repair b. Secondary intention c. Tertiary intention d. Primary intention

ANS: D A clean surgical incision is an example of a wound with little loss of tissue that heals with primary intention. The skin edges are approximated or closed, and the risk for infection is low. Partial-thickness repairs are done on partial-thickness wounds that are shallow, involving loss of the epidermis and maybe partial loss of the dermis. These wounds heal by regeneration because the epidermis regenerates. Tertiary intention is seen when a wound is left open for several days, and then the wound edges are approximated. Wound closure is delayed until the risk of infection is resolved. A wound involving loss of tissue such as a burn or a pressure ulcer or laceration heals by secondary intention. The wound is left open until it becomes filled with scar tissue. It takes longer for a wound to heal by secondary intention; thus the chance of infection is greater.

The nurse is caring for a patient who has experienced a total abdominal hysterectomy. Which nursing observation of the incision will indicate the patient is experiencing a complication of wound healing? a. The site is hurting. b. The site is approximated. c. The site has started to itch. d. The site has a mass, bluish in color.

ANS: D A hematoma is a localized collection of blood underneath the tissues. It appears as swelling, change in color, sensation, or warmth or a mass that often takes on a bluish discoloration. A hematoma near a major artery or vein is dangerous because it can put pressure on the vein or artery and obstruct blood flow. Itching is not a complication. Incisions should be approximated with edges together; this is a sign of normal healing. After surgery, when nerves in the skin and tissues have been traumatized by the surgical procedure, it is expected that the patient will experience pain.

A nurse is completing an assessment of the patient. Which principle is a priority? a. Foot care will always be important. b. Daily bathing will always be important. c. Hygiene needs will always be important. d. Critical thinking will always be important.

ANS: D A patient's condition is always changing, requiring ongoing critical thinking and changing of nursing diagnoses. Apply the elements of critical thinking as you use the nursing process to meet patients' hygiene needs. Critical thinking will help you determine when foot care, daily bathing, and hygiene needs are important and when they are not.

A nurse is assessing a patient's wound. Which nursing observation will indicate the wound healed by secondary intention? a. Minimal loss of tissue function b. Permanent dark redness at site c. Minimal scar tissue d. Scarring that may be severe

ANS: D A wound healing by secondary intention takes longer than one healing by primary intention. The wound is left open until it becomes filled with scar tissue. If the scarring is severe, permanent loss of function often occurs. Wounds that heal by primary intention heal quickly with minimal scarring. Scar tissue contains few pigmented cells and has a lighter color than normal skin.

The nurse is caring for a patient with a Stage II pressure ulcer and has assigned a nursing diagnosis of Risk for infection. The patient is unconscious and bedridden. The nurse is completing the plan of care and is writing goals for the patient. Which is the best goal for this patient? a. The patient will state what to look for with regard to an infection. b. The patient's family will demonstrate specific care of the wound site. c. The patient's family members will wash their hands when visiting the patient. d. The patient will remain free of odorous or purulent drainage from the wound.

ANS: D Because the patient has an open wound and the skin is no longer intact to protect the tissue, the patient is at increased risk for infection. The nurse will be assessing the patient for signs and symptoms of infection, including an increase in temperature, an increase in white count, and odorous and purulent drainage from the wound. The patient is unconscious and is unable to communicate the signs and symptoms of infection. It is important for the patient's family to be able to demonstrate how to care for the wound and wash their hands, but these statements are not goals or outcomes for this nursing diagnosis.

The nurse is caring for a surgical patient. Which intervention is most important for the nurse to complete to decrease the risk of pressure ulcers and encourage the patient's willingness and ability to increase mobility? a. Explain the risks of immobility to the patient. b. Turn the patient every 3 hours while in bed. c. Encourage the patient to sit up in the chair. d. Provide analgesic medication as ordered.

ANS: D Maintaining adequate pain control (providing analgesic medications) and patient comfort increases the patient's willingness and ability to increase mobility, which in turn reduces pressure ulcer risks. Although sitting in the chair is beneficial, it does not increase mobility or provide pain control. Explaining the risk of immobility is important for the patient because it may impact the patient's willingness but not his or her ability. Turning the patient is important for decreasing pressure ulcers but needs to be done every 2 hours and, again, does not influence the patient's ability to increase mobility.

A nurse is preparing a patient for a magnetic resonance imaging (MRI) scan. Which nursing action is most important? a. Ensuring that the patient does not eat or drink 2 hours before the examination. b. Administering a colon cleansing product 6 hours before the examination. c. Obtaining an order for a pain medication before the test is performed. d. Removing all of the patient's metallic jewelry.

ANS: D No jewelry or metal products should be in the same room as an MRI machine because of the high-power magnet used in the machine. The patient needs to be NPO 4 to 6 hours before the examination. Colon cleansing products are not necessary for MRIs. Pain medication is not needed before the examination is performed.

The patient has a risk for skin impairment and has a 15 on the Braden Scale upon admission. The nurse has implemented interventions. Upon reassessment, which Braden score will be the best sign that the risk for skin breakdown is removed? a. 12 b. 13 c. 20 d. 23

ANS: D The best sign is a perfect score of 23. The Braden Scale is composed of six subscales: sensory perception, moisture, activity, mobility, nutrition, and friction and shear. The total score ranges from 6 to 23, and a lower total score indicates a higher risk for pressure ulcer development. The cutoff score for onset of pressure ulcer risk with the Braden Scale in the general adult population is 18.

The patient has the nursing diagnosis of Impaired physical mobility related to pain in the left shoulder. Which priority action will the nurse take? a. Encourage the patient to do self-care. b. Keep the patient as mobile as possible. c. Encourage the patient to perform ROM. d. Assist the patient with comfort measures.

ANS: D The diagnosis related to pain requires the nurse to assist the patient with comfort measures so that the patient is then willing and more able to move. Pain must be controlled so the patient will not be reluctant to initiate movement. The diagnosis related to reluctance to initiate movement requires interventions aimed at keeping the patient as mobile as possible and encouraging the patient to perform self-care and ROM.

A nursing assistive personnel (NAP) is providing AM care to patients. Which action by the NAP will require the nurse to intervene? a. Not offering a backrub to a patient with fractured ribs b. Not offering to wash the hair of a patient with neck trauma c. Turning off the television while giving a backrub to the patient d. Turning patient's head with neck injury to side when giving oral care

ANS: D The nurse must intervene if the NAP turns the patient's head with a neck injury; this is contraindicated and must be stopped to prevent further injury. All the other actions are appropriate and do not need follow-up. Consult the medical record for any contraindications to a massage (e.g., fractured ribs, burns, and heart surgery). Before washing a patient's hair, determine that there are no contraindications to procedure (e.g., neck injury). When providing a backrub, enhance relaxation by reducing noise (turning off the television) and ensuring that the patient is comfortable.

The patient is being treated for cancer with weekly radiation therapy to the head and chemotherapy treatments. Which assessment is the priority? a. Feet b. Nail beds c. Perineum d. Oral cavity

ANS: D The oral cavity is the priority. Radiation to the head reduces salivary flow and lowers pH of saliva, leading to stomatitis and tooth decay, while chemotherapy drugs kill the normal cells lining the oral cavity, leading to ulcers and inflammation. While the feet, nail beds, and perineum are important, they are not as affected as the oral cavity with head or neck radiation and chemotherapy.

The nurse is caring for a patient after an open abdominal aortic aneurysm repair. The nurse requests an abdominal binder and carefully applies the binder. Which is the best explanation for the nurse to use when teaching the patient the reason for the binder? a. It reduces edema at the surgical site. b. It secures the dressing in place. c. It immobilizes the abdomen. d. It supports the abdomen.

ANS: D The patient has a large abdominal incision. This incision will need support, and an abdominal binder will support this wound, especially during movement, as well as during deep breathing and coughing. A binder can be used to immobilize a body part (e.g., an elastic bandage applied around a sprained ankle). A binder can be used to prevent edema, for example, in an extremity but in this case is not used to reduce edema at a surgical site. A binder can be used to secure dressings such as elastic webbing applied around a leg after vein stripping.

The nurse is managing bowel training for a patient. To which patient is the nurse most likely providing care? a. A 25-year-old patient with diarrhea b. A 30-year-old patient with Clostridium difficile c. A 40-year-old patient with an ileostomy d. A 70-year-old patient with stool incontinence

ANS: D The patient with chronic constipation or fecal incontinence secondary to cognitive impairment may benefit from bowel training, also called habit training. An ileostomy, diarrhea, and C. difficile all relate to uncontrollable bowel movements, for which no method can be used to set up a schedule of elimination.

A patient recovering from a leg fracture after a fall reports having dull pain in the affected leg and rates it as a 7 on a 0 to 10 scale. The patient is not able to walk around in the room with crutches because of leg discomfort. Which nursing intervention is priority? a. Assist the patient to walk in the room with crutches. b. Obtain a walker for the patient. c. Consult physical therapy. d. Administer pain medication.

ANS: D The patient's pain is a 7, indicating the priority is pain relief (administer pain medication). Acute pain is the priority because the nurse can address the problem of immobility after the patient receives adequate pain relief. Assisting the patient to walk or obtaining a walker will not address the pain the patient is experiencing.

The nurse is caring for an older-adult patient with a diagnosis of urinary tract infection (UTI). Upon assessment the nurse finds the patient confused and agitated. How will the nurse interpret these assessment findings? a. These are normal signs of aging. b. These are early signs of dementia. c. These are purely psychological in origin. d. These are common manifestation with UTIs.

ANS: D The primary symptom of compromised older patients with an acute urinary tract infection or fever is confusion. Acute confusion in older adults is not normal; a thorough nursing assessment is the priority. With the diagnosis of urinary tract infection, these are not early signs of dementia and they are not purely psychological.

The patient is unable to move self and needs to be pulled up in bed. What will the nurse do to make this procedure safe? a. Place the pillow under the patient's head and shoulders. b. Do by self if the bed is in the flat position. c. Place the side rails in the up position. d. Use a friction-reducing device.

ANS: D This is not a one-person task. Helping a patient move up in bed without help from other co-workers or without the aid of an assistive device (e.g., friction-reducing pad) is not recommended and is not considered safe for the patient or the nurse. Remove the pillow from under head and shoulders and place it at the head of the bed to prevent striking the patient's head against the head of the bed. When pulling a patient up in bed, the bed should be flat to gain gravity assistance, and the side rails should be down.

The nurse is completing a skin assessment on a patient with darkly pigmented skin. Which item should the nurse use first to assist in staging an ulcer on this patient? a. Disposable measuring tape b. Cotton-tipped applicator c. Sterile gloves d. Halogen light

ANS: D When assessing a patient with darkly pigmented skin, proper lighting is essential to accurately complete the first step in assessment—inspection—and the entire assessment process. Natural light or a halogen light is recommended. Fluorescent light sources can produce blue tones on darkly pigmented skin and can interfere with an accurate assessment. Other items that could possibly be used during the assessment include gloves for infection control, a disposable measuring device to measure the size of the wound, and a cotton-tipped applicator to measure the depth of the wound, but these items are not the first items used.

A patient is admitted with a stroke. The outcome of this disorder is uncertain, but the patient is unable to move the right arm and leg. The nurse starts passive range-of-motion (ROM) exercises. Which finding indicates successful goal achievement? a. Heart rate decreased. b. Contractures developed. c. Muscle strength improved. d. Joint mobility maintained.

ANS: D When patients cannot participate in active ROM, maintain joint mobility and prevent contractures by implementing passive ROM into the plan of care. Exercise and active ROM can improve muscle strength. ROM is not performed for the heart but for the joints.

The nurse is providing oral care to an unconscious patient. Which action should the nurse take? a. Moisten the mouth using lemon-glycerin sponges. b. Hold the patient's mouth open with gloved fingers. c. Use foam swabs to help remove plaque. d. Suction the oral cavity.

ANS: D When providing oral hygiene to an unconscious patient, the nurse needs to protect him or her from choking and aspiration. Have two nurses provide care; one nurse does the actual cleaning, and the other caregiver removes secretions with suction equipment. The nurse can delegate nursing assistive personnel to participate. Some agencies use equipment that combines a mouth swab with the suction device. This device can be used safely by one nurse to provide oral care. Commercially made foam swabs are ineffective in removing plaque. Do not use lemon-glycerin sponges because they dry mucous membranes and erode tooth enamel. While cleansing the oral cavity, use a small oral airway or a padded tongue blade to hold the mouth open. Never use your fingers to hold the patient's mouth open. A human bite contains multiple pathogenic microorganisms.

A patient expresses concerns over having black stool. The fecal occult test is negative. Which response by the nurse is most appropriate? a. "This is probably a false negative; we should rerun the test." b. "You should schedule a colonoscopy as soon as possible." c. "Are you under a lot of stress?" d. "Do you take iron supplements?"

ANS: D Certain medications and supplements, such as iron, can alter the color of stool (black or tarry). Since the fecal occult test is negative, bleeding is not occurring. The fecal occult test takes three separate samples over a period of time and is a fairly reliable test. A colonoscopy is health prevention screening that should be done every 5 to 10 years; it is not the nurse's initial priority. Stress alters GI motility and stool consistency, not color.

Which action by the nurse will be the most important for preventing skin impairment in a mobile patient with local nerve damage? a. Insert an indwelling urinary catheter. b. Limit caloric and protein intake. c. Turn the patient every 2 hours. d. Assess for pain during a bath.

ANS: D During a bath, assess the status of sensory nerve function by checking for touch, pain, heat, cold, and pressure. When restricted from moving freely, dependent body parts are exposed to pressure that reduces circulation. However, this patient is mobile and therefore is able to change positions. Limiting caloric and protein intake may result in impaired or delayed wound healing. A mobile patient can use bathroom facilities or a urinal and does not need a urinary catheter.

A nurse is providing range of motion to the shoulder and must perform external rotation. Which action will the nurse take? a. Moves patient's arm in a full circle b. Moves patient's arm cross the body as far as possible c. Moves patient's arm behind body, keeping elbow straight d. Moves patient's arm until thumb is upward and lateral to head with elbow flexed

ANS: D External rotation: With elbow flexed, move arm until thumb is upward and lateral to head. Circumduction: Move arm in full circle (Circumduction is combination of all movements of ball-and-socket joint.) Adduction: Lower arm sideways and across body as far as possible. Hyperextension: Move arm behind body, keeping elbow straight.

A nurse is developing an exercise plan for a middle-aged patient. In which order will the nurse instruct the patient to execute the plan, beginning with the first step? 1. Design the fitness program. 2. Assemble equipment. 3. Assess fitness level. 4. Monitor progress. 5. Get started. a. 5, 1, 3, 2, 4 b. 1, 2, 3, 5, 4 c. 2, 5, 3, 1, 4 d. 3, 1, 2, 5, 4

ANS: D Five steps to beginning an exercise program are Step 1: Assess fitness level; Step 2: Design the fitness program; Step 3: Assemble equipment; Step 4: Get started; and Step 5: Monitor progress.

The nurse is providing education about the importance of proper foot care to a patient who has diabetes mellitus. Which primary goal is the nurse trying to achieve? a. Prevention of plantar warts b. Prevention of foot fungus c. Prevention of neuropathy d. Prevention of amputation

ANS: D Foot ulceration is the most common single precursor to lower extremity amputations among persons with diabetes. Prevention of plantar warts and foot fungus are important but not the primary goal. Neuropathy is a degeneration of the peripheral nerves usually due to poor control of blood glucose levels; it is not a direct result of foot care.

The nurse is admitting a patient who has been diagnosed as having had a stroke. The health care provider writes orders for "ROM as needed." What should the nurse do next? a. Restrict patient's mobility as much as possible. b. Realize the patient is unable to move extremities. c. Move all the patient's extremities. d. Further assess the patient.

ANS: D Further assessment of the patient is needed to determine what the patient is able to perform. Some patients are able to move some joints actively, whereas the nurse passively moves others. With a weak patient, the nurse may have to support an extremity while the patient performs the movement. In general, exercises need to be as active as health and mobility allow.

The nurse is observing the way a patient walks. Which aspect is the nurse assessing? a. Activity tolerance b. Body alignment c. Range of motion d. Gait

ANS: D Gait describes a particular manner or style of walking. Activity tolerance is the type and amount of exercise or work that a person is able to perform. Body alignment refers to the position of the joints, tendons, ligaments, and muscles while standing, sitting, and lying. Range of motion is the maximum amount of movement available at a joint in one of the three planes of the body: sagittal, frontal, or transverse.

A nurse is performing passive range of motion (ROM) and splinting on an at-risk patient. Which finding will indicate goal achievement for the nurse's action? a. Prevention of atelectasis b. Prevention of renal calculi c. Prevention of pressure ulcers d. Prevention of joint contractures

ANS: D Goal achievement for passive ROM is prevention of joint contractures. Contractures develop in joints not moved periodically through their full ROM. ROM exercises reduce the risk of contractures. Researchers noted that prompt use of splinting with prescribed ROM exercises reduced contractures and improved active range of joint motion in affected lower extremities. Deep breathing and coughing and using an incentive spirometer will help prevent atelectasis. Adequate hydration helps prevent renal calculi and urinary tract infections. Interventions aimed at prevention of pressure ulcers include positioning, skin care, and the use of therapeutic devices to relieve pressure.

The nurse is caring for a postoperative medial meniscus repair of the right knee. Which action should the nurse take to assist with pain management? a. Monitor vital signs every 15 minutes. b. Check pulses in the right foot. c. Keep the leg dependent. d. Apply ice.

ANS: D Ice assists in preventing edema formation, controlling bleeding, and anesthetizing the body part. Elevation (not dependent) assists in preventing edema, which in turn can cause pain. Monitoring vital signs every 15 minutes is routine postoperative care and includes a pain assessment but in itself is not an intervention that decreases pain. Checking the pulses is important to monitor the circulation of the extremity but in itself is not a pain management intervention.

A nurse is caring for an immobile patient. Which metabolic alteration will the nurse monitor for in this patient? a. Increased appetite b. Increased diarrhea c. Increased metabolic rate d. Altered nutrient metabolism

ANS: D Immobility disrupts normal metabolic functioning: decreasing the metabolic rate, altering the metabolism of carbohydrates, fats, and proteins; causing fluid, electrolyte, and calcium imbalances; and causing gastrointestinal disturbances such as decreased appetite and slowing of peristalsis, leading to constipation.

A nurse is caring for a patient who has some immobility from noninflammatory joint degeneration. The nurse is teaching the patient about this process. Which information will the nurse include in the teaching session? a. This will affect synovial fluid. b. This will affect the body systemically. c. This involves mostly non-weight-bearing joints. d. This involves overgrowth of bone at the articular ends.

ANS: D Joint degeneration, which can occur with inflammatory and noninflammatory disease, is marked by changes in articular cartilage combined with overgrowth of bone at the articular ends. Degenerative changes commonly affect weight-bearing joints. Synovial fluid is normal in noninflammatory diseases. Inflammatory joint disease (e.g., arthritis) is characterized by inflammation or destruction of the synovial membrane and articular cartilage and by systemic signs of inflammation.

A nurse is preparing a care plan for a patient who is immobile. Which psychosocial aspect will the nurse consider? a. Loss of bone mass b. Loss of strength c. Loss of weight d. Loss of hope

ANS: D Loss of hope is a psychosocial aspect. Patients with restricted mobility may have some depression. Depression is an affective disorder characterized by exaggerated feelings of sadness, melancholy, dejection, worthlessness, emptiness, and hopelessness out of proportion to reality. All the rest are physiological aspects: bone mass, strength, and weight.

A patient with a hip fracture is having difficulty defecating into a bedpan while lying in bed. Which action by the nurse will assist the patient in having a successful bowel movement? a. Preparing to administer a barium enema b. Withholding narcotic pain medication c. Administering laxatives to the patient d. Raising the head of the bed

ANS: D Lying in bed is an unnatural position; raising the head of the bed assists the patient into a more normal position that allows proper contraction of muscles for elimination. Laxatives would not give the patient control over bowel movements. A barium enema is a diagnostic test, not an intervention to promote defecation. Pain relief measures should be given; however, preventative action should be taken to prevent constipation.

A nurse is working in a facility that uses no-lift policies. Which benefits will the nurse observe in the facility? (Select all that apply.) a. Reduced number of work-related injuries b. Increased musculoskeletal accidents c. Reduced safety of patients d. Improved health of nurses e. Increased indirect costs

ANS: A, D Implementing evidence-based interventions and programs (e.g., lift teams) reduces the number of work-related injuries, which improves the health of the nurse and reduces indirect costs to the health care facility (e.g., workers' compensation and replacing injured workers). Knowing the movements and functions of muscles in maintaining posture and movement and implementing evidence-based knowledge about safe patient handling are essential to protecting the safety of both the patient and the nurse.

Upon assessment a nurse discovers that a patient has erythema. Which actions will the nurse take? (Select all that apply.) a. Consult a dietitian. b. Increase fiber in the diet. c. Place on chest physiotherapy. d. Increase frequency of turning. e. Place on pressure-relieving mattress.

ANS: A, D, E If skin shows areas of erythema and breakdown, increase the frequency of turning and repositioning; place the turning schedule above the patient's bed; implement other activities per agency skin care policy or protocol (e.g., assess more frequently, consult dietitian, place patient on pressure-relieving mattress). Increased fiber will help constipation. Chest physiotherapy is for respiratory complications.

The patient is being fitted with a hearing aid. In teaching the patient how to care for the hearing aid, which instructions will the nurse provide? a. Change the battery every day or as needed. b. Adjust the volume for a talking distance of 1 yard. c. Wear the hearing aid 24 hours per day except when sleeping. d. Avoid the use of hairspray, but aerosol perfumes are allowed.

ANS: B Adjust volume to a comfortable level for talking at a distance of 1 yard. Initially, wear a hearing aid for 15 to 20 minutes; then gradually increase wear time to 10 to 12 hours per day. Batteries last 1 week with daily wearing of 10 to 12 hours. Avoid the use of hairspray and perfume while wearing hearing aids. Residue from the spray can cause the aid to become oily and greasy.

A patient presents to the emergency department with a laceration of the right forearm caused by a fall. After determining that the patient is stable, what is the next best step for the nurse to take? a. Inspect the wound for foreign bodies. b. Inspect the wound for bleeding. c. Determine the size of the wound. d. Determine the need for a tetanus antitoxin injection.

ANS: B After determining that a patient's condition is stable, inspect the wound for bleeding. An abrasion will have limited bleeding, a laceration can bleed more profusely, and a puncture wound bleeds in relation to the size and depth of the wound. Address any bleeding issues. Inspect the wound for foreign bodies; traumatic wounds are dirty and may need to be addressed. Determine the size of the wound. A large open wound may expose bone or tissue and be protected, or the wound may need suturing. When the wound is caused by a dirty penetrating object, determine the need for a tetanus vaccination.

The nurse is caring for a group of patients. Which task can the nurse delegate to the nursing assistive personnel? a. Assessing a surgical patient for risk of pressure ulcers b. Applying an elastic bandage to a medical-surgical patient c. Treating a pressure ulcer on the buttocks of a medical patient d. Implementing negative-pressure wound therapy on a stable patient

ANS: B Applying an elastic bandage to a medical-surgical patient can be delegated to the nursing assistive personnel (NAP). Assessing pressure ulcer risk, treating a pressure ulcer, and implementing negative-pressure wound therapy cannot be delegated to an NAP.

The nurse is assessing body alignment for a patient who is immobilized. Which patient position will the nurse use? a. Supine position b. Lateral position c. Lateral position with positioning supports d. Supine position with no pillow under the patient's head

ANS: B Assess body alignment for a patient who is immobilized or bedridden with the patient in the lateral position, not supine. Remove all positioning supports from the bed except for the pillow under the head, and support the body with an adequate mattress.

The nurse is emptying an ileostomy pouch for a patient. Which assessment finding will the nurse report immediately? a. Liquid consistency of stool b. Presence of blood in the stool c. Malodorous stool d. Continuous output from the stoma

ANS: B Blood in the stool indicates a problem, and the health care provider should be notified. All other options are expected findings for an ileostomy. The stool should be liquid, there should be an odor, and the output should be continuous.

The nurse is teaching the patient about flossing and oral hygiene. Which instruction will the nurse include in the teaching session? a. Using waxed floss prevents bleeding. b. Flossing removes plaque and tartar from the teeth. c. Performing flossing at least 3 times a day is beneficial. d. Applying toothpaste to the teeth before flossing is harmful.

ANS: B Dental flossing removes plaque and tartar between teeth. To prevent bleeding, the patient should use unwaxed floss. Flossing once a day is sufficient. If toothpaste is applied to the teeth before flossing, fluoride will come in direct contact with tooth surfaces, aiding in cavity prevention.

The nurse is caring for a patient with a wound. The patient appears anxious as the nurse is preparing to change the dressing. Which action should the nurse take? a. Turn on the television. b. Explain the procedure. c. Tell the patient "Close your eyes." d. Ask the family to leave the room.

ANS: B Explaining the procedure educates the patient regarding the dressing change and involves him in the care, thereby allowing the patient some control in decreasing anxiety. Telling the patient to close the eyes and turning on the television are distractions that do not usually decrease a patient's anxiety. If the family is a support system, asking support systems to leave the room can actually increase a patient's anxiety.

The nurse is intervening for a family member with role strain. Which direct care nursing intervention is most appropriate? a. Assisting with activities of daily living b. Counseling about respite care options c. Teaching range-of-motion exercises d. Consulting with a social worker

ANS: B Family caregivers need assistance in adjusting to the physical and emotional demands of caregiving. Sometimes they need respite (i.e., a break from providing care). Counseling is an example of a direct care nursing intervention. The other options do not address the identified problem of role strain (activities of daily living and range-of-motion exercises). Consulting is an indirect care nursing intervention.

The nurse is providing oral care to an unconscious patient and notes that the patient has extremely bad breath. Which term will the nurse use when reporting to the oncoming shift? a. Cheilitis b. Halitosis c. Glossitis d. Dental caries

ANS: B Halitosis is the term for "bad breath." Cheilitis is the term for cracked lips. Dental caries are cavities in the teeth and could be a cause of the halitosis. Glossitis is the term for inflamed tongue.

When providing hygiene for an older-adult patient, the nurse closely assesses the skin. What is the rationale for the nurse's action? a. Outer skin layer becomes more resilient. b. Less frequent bathing may be required. c. Skin becomes less subject to bruising. d. Sweat glands become more active.

ANS: B In older adults, daily bathing as well as bathing with water that is too hot or soap that is harsh causes the skin to become excessively dry. As the patient ages, the skin thins and loses its resiliency and moisture, and lubricating skin glands become less active, making the skin fragile and prone to bruising and breaking.

The nurse establishes trust and talks with a school-aged patient before administering an injection. Which type of implementation skill is the nurse using? a. Cognitive b. Interpersonal c. Psychomotor d. Judgmental

ANS: B Nursing practice includes cognitive, interpersonal, and psychomotor skills. Interpersonal skills involve developing trusting relationships with patients, conveying caring and compassion, and communicating clearly. Cognitive skills include critical thinking and decision-making skills. Psychomotor skill requires the integration of cognitive and motor abilities, such as administering the injection. Being judgmental is not appropriate in nursing; nurses are nonjudgmental.

The nurse is caring for a patient with inner ear problems. Which goal is the priority? a. Maintain balance. b. Maintain proprioception. c. Maintain muscle strength. d. Maintain body alignment.

ANS: A Within the inner ear are the semicircular canals, three fluid-filled structures that help maintain balance. Proprioception is the awareness of the position of the body and its parts, and proprioceptors are located on nerve endings, not the inner ear. Muscle strength is maintained with activity and exercise. Although body alignment is important, it is not maintained by the inner ear.

The nurse is collaborating with the dietitian about a patient with a Stage III pressure ulcer. Which nutrient will the nurse most likely increase after collaboration with the dietitian? a. Fat b. Protein c. Vitamin E d. Carbohydrate

ANS: B Protein needs are especially increased in supporting the activity of wound healing. The physiological processes of wound healing depend on the availability of protein, vitamins (especially A and C), and the trace minerals of zinc and copper. Wound healing does not require increased amounts of fats or carbohydrates. Vitamin E will not be increased for wound healing.

The nurse is caring for a patient with a wound healing by full-thickness repair. Which phases will the nurse monitor for in this patient? (Select all that apply.) a. Hemostasis b. Maturation c. Inflammatory d. Proliferative e. Reproduction f. Reestablishment of epidermal layers

ANS: A, B, C, D The four phases involved in the healing process of a full-thickness wound are hemostasis, inflammatory, proliferative, and maturation. Three components are involved in the healing process of a partial-thickness wound: inflammatory response, epithelial proliferation (reproduction) and migration, and reestablishment of the epidermal layers.

The nurse is caring for a patient who has peripheral neuropathy. Which clinical manifestations does the nurse expect to find upon assessment? (Select all that apply.) a. Abnormal gait b. Foot deformities c. Absent or decreased pedal pulses d. Muscle wasting of lower extremities e. Decreased hair growth on legs and feet

ANS: A, B, D A patient with peripheral neuropathy has muscle wasting of lower extremities, foot deformities, and abnormal gait. A patient with vascular insufficiency will have decreased hair growth on legs and feet, absent or decreased pulses, and thickened nails.

The nurse is providing oral care to a patient. In which order will the nurse clean the oral cavity, starting with the first area? 1. Roof of mouth, gums, and inside cheek 2. Chewing and inner tooth surfaces 3. Outer tooth surfaces 4. Tongue a. 4, 1, 3, 2 b. 3, 2, 4, 1 c. 2, 3, 1, 4 d. 1, 4, 2, 3

ANS: C Oral care is provided in the following sequence: Clean chewing and inner tooth surfaces first. Clean outer tooth surfaces. Moisten brush with chlorhexidine rinse to rinse. Use toothette to clean roof of mouth, gums, and inside cheeks. Gently brush tongue but avoid stimulating gag reflex. Rinse.

A nurse is caring for a postoperative patient. Which finding will alert the nurse to a potential wound dehiscence? a. Protrusion of visceral organs through a wound opening b. Chronic drainage of fluid through the incision site c. Report by patient that something has given way d. Drainage that is odorous and purulent

ANS: C Patients often report feeling as though something has given way with dehiscence. Dehiscence occurs when an incision fails to heal properly and the layers of skin and tissue separate. It involves abdominal surgical wounds and occurs after a sudden strain such as coughing, vomiting, or sitting up in bed. Evisceration is seen when vital organs protrude through a wound opening. When there is an increase in serosanguineous drainage from a wound in the first few days after surgery, be alert for the potential for dehiscence. Infection is characterized by drainage that is odorous and purulent.

After performing foot care, the nurse checks the medical record and discovers that the patient has a foot disorder caused by a virus. Which condition did the nurse most likely observe? a. Corns b. A callus c. Plantar warts d. Athlete's foot

ANS: C Plantar warts appear on the sole of the foot and are caused by the papillomavirus. Corns are caused by friction and pressure from ill-fitting or loose shoes. Athlete's foot (tinea pedis) is a fungal infection and can spread to other body parts. A callus is caused by local friction or pressure.

The nurse is providing care to a patient who is bedridden. The nurse raises the height of the bed. What is the rationale for the nurse's action? a. Narrows the nurse's base of support. b. Allows the nurse to bring feet closer together. c. Prevents a shift in the nurse's base of support. d. Shifts the nurse's center of gravity farther away from the base of support.

ANS: C Raising the height of the bed when performing a procedure prevents bending too far at the waist and shifting the base of support. Balance is maintained by proper body alignment and posture through two simple techniques. First, widen the base of support by separating the feet to a comfortable distance. Second, increase balance by bringing the center of gravity closer to the base of support.

The nurse is providing perineal care to an uncircumcised male patient. Which action will the nurse take? a. Leave the foreskin alone because there is little chance of infection. b. Retract the foreskin for cleansing and allow it to return on its own. c. Retract the foreskin and return it to its natural position when done. d. Leave the foreskin retracted.

ANS: C Return the foreskin to its natural position. Keeping the foreskin retracted leads to tightening of the foreskin around the shaft of the penis, causing local edema and discomfort. The foreskin may not return to its natural position on its own. Patients at greatest risk for infection are uncircumcised males.

A nurse is following the no-lift policy when working to prevent personal injury. Which type of personal back injury is the nurse most likely trying to prevent? a. Thoracic b. Cervical c. Lumbar d. Sacral

ANS: C The most common back injury for nurses is strain on the lumbar muscle group, which includes the muscles around the lumbar vertebrae. While cervical, thoracic, and sacral can occur, lumbar is the most common.

The patient has been in bed for several days and needs to be ambulated. Which action will the nurse take first? a. Maintain a narrow base of support. b. Dangle the patient at the bedside. c. Encourage isometric exercises. d. Suggest a high-calcium diet.

ANS: B To prevent injury, nurses implement interventions that reduce or eliminate the effects of orthostatic hypotension. Mobilize the patient as soon as the physical condition allows, even if this only involves dangling at the bedside or moving to a chair. A wide base of support increases balance. Isometric exercises (i.e., activities that involve muscle tension without muscle shortening) have no beneficial effect on preventing orthostatic hypotension, but they improve activity tolerance. A high-calcium diet can help with osteoporosis but can be detrimental in an immobile patient.

Which patient will the nurse assess most closely for an ileus? a. A patient with a fecal impaction b. A patient with chronic cathartic abuse c. A patient with surgery for bowel disease and anesthesia d. A patient with suppression of hydrochloric acid from medication

ANS: C Any surgery that involves direct manipulation of the bowel temporarily stops peristalsis. Anesthesia can also cause cessation of peristalsis. This condition, called an ileus, usually lasts about 24 to 48 hours. Fecal impaction, cathartic abuse, and medication to suppress hydrochloric acid will have bowel sounds, but they may be hypoactive or hyperactive.

The nurse is planning care for a group of patients. Which task will the nurse assign to the nursing assistive personnel (NAP)? a. Performing the first postoperative pouch change b. Maintaining a nasogastric tube c. Administering an enema d. Digitally removing stool

ANS: C The skill of administering an enema can be delegated to an NAP. The skill of inserting and maintaining a nasogastric (NG) tube cannot be delegated to an NAP. The nurse should do the first postoperative pouch change. Digitally removing stool cannot be delegated to nursing assistive personnel.

The nurse will anticipate which diagnostic examination for a patient with black tarry stools? a. Ultrasound b. Barium enema c. Endoscopy d. Anorectal manometry

ANS: C Black tarry stools are an indication of bleeding in the GI tract; endoscopy would allow visualization of the bleeding. No other option (ultrasound, barium enema, and anorectal manometry) would allow GI visualization.

A nurse is assisting a patient in making dietary choices that promote healthy bowel elimination. Which menu option should the nurse recommend? a. Broccoli and cheese soup with potato bread b. Turkey and mashed potatoes with brown gravy c. Grape and walnut chicken salad sandwich on whole wheat bread d. Dinner salad topped with hard-boiled eggs, cheese, and fat-free dressing

ANS: C Grapes and whole wheat bread are high fiber and should be chosen. Cheese, eggs, potato bread, and mashed potatoes do not contain as much fiber as whole wheat bread. A healthy diet for the bowel should include foods high in bulk-forming fiber. Whole grains, fresh fruit, and fresh vegetables are excellent sources. Foods without much fiber and with high levels of fat can slow down peristalsis, causing constipation.

A nurse is providing passive range of motion (ROM) for a patient with impaired mobility. Which technique will the nurse use for each movement? a. Each movement is repeated 5 times by the patient. b. Each movement is performed until the patient experiences pain. c. Each movement is completed quickly and smoothly by the nurse. d. Each movement is moved just to the point of resistance by the nurse.

ANS: D Passive ROM exercises are performed by the nurse. Carry out movements slowly and smoothly, just to the point of resistance; ROM should not cause pain. Never force a joint beyond its capacity. Each movement needs to be repeated 5 times during the session. The patient moves all joints through ROM unassisted in active ROM.

The nurse is caring for a patient with Clostridium difficile. Which nursing actions will have the greatest impact in preventing the spread of the bacteria? a. Appropriate disposal of contaminated items in biohazard bags b. Monthly in-services about contact precautions c. Mandatory cultures on all patients d. Proper hand hygiene techniques

ANS: D Proper hand hygiene is the best way to prevent the spread of bacteria. Soap and water are mandatory. Monthly in-services place emphasis on education, not on action. Biohazard bags are appropriate but cannot be used on every item that C. difficile comes in contact with, such as a human. Mandatory cultures are expensive and unnecessary and would not prevent the spread of bacteria.

The nurse is teaching a health class about the gastrointestinal tract. The nurse will explain that which portion of the digestive tract absorbs most of the nutrients? a. Ileum b. Cecum c. Stomach d. Duodenum

ANS: D The duodenum and jejunum absorb most nutrients and electrolytes in the small intestine. The ileum absorbs certain vitamins, iron, and bile salts. Food is broken down in the stomach. The cecum is the beginning of the large intestine.

A patient has a fecal impaction. Which portion of the colon will the nurse assess? a. Descending b. Transverse c. Ascending d. Rectum

ANS: D A fecal impaction is a collection of hardened feces wedged in the rectum that cannot be expelled. It results from unrelieved constipation. Feces at this point in the colon contain the least amount of moisture. Feces found in the ascending, transverse, and descending colon still consist mostly of liquid and do not form a hardened mass.

Which action should the nurse take first during the initial phase of implementation? a. Determine patient outcomes and goals. b. Prioritize patient's nursing diagnoses. c. Evaluate interventions. d. Reassess the patient.

ANS: D Assessment is a continuous process that occurs each time the nurse interacts with a patient. During the initial phase of implementation, reassess the patient. Determining the patient's goals and prioritizing diagnoses take place in the planning phase before choosing interventions. Evaluation is the last step of the nursing process.

A staff development nurse is providing an inservice for other nurses to educate them about the Nursing Interventions Classification (NIC) system. During the inservice, which statement made by one of the nurses in the room requires the staff development nurse to clarify the information provided? a. "This system can help medical students determine the cost of the care they provide to patients." b. "If the nursing department uses this system, communication among nurses who work throughout the hospital may be enhanced." c. "We could use this system to help organize orientation for new nursing employees because we can better explain the nursing interventions we use most frequently on our unit." d. "The NIC system provides one way to improve safe and effective documentation in the hospital's electronic health record."

ANS: A NIC does not help determine the cost of services provided by nurses. The staff development nurse would need to correct this misconception. Because this system is specific to nursing practice, it would not help medical students determine the costs of care. The NIC system developed by the University of Iowa differentiates nursing practice from that of other health care disciplines. All the other statements are true. Benefits of using NIC include enhancing communication among nursing staff and documentation, especially within health information systems such as an electronic documentation system. NIC also helps nurses identify the nursing interventions they implement most frequently. Units that identify routine nursing interventions can use this information to develop checklists for orientation.

The nurse is caring for an older-adult patient who has been diagnosed with a stroke. Which intervention will the nurse add to the care plan? a. Encourage the patient to perform as many self-care activities as possible. b. Provide a complete bed bath to promote patient comfort. c. Coordinate with occupational therapy for gait training. d. Place the patient on bed rest to prevent fatigue.

ANS: A Nurses should encourage the older-adult patient to perform as many self-care activities as possible, thereby maintaining the highest level of mobility. Sometimes nurses inadvertently contribute to a patient's immobility by providing unnecessary help with activities such as bathing and transferring. Placing the patient on bed rest without sufficient ambulation leads to loss of mobility and functional decline, resulting in weakness, fatigue, and increased risk for falls. After a stroke or brain attack, a patient likely receives gait training from a physical therapist; speech rehabilitation from a speech therapist; and help from an occupational therapist for ADLs such as dressing, bathing and toileting, or household chores.

A patient with a fecal impaction has an order to remove stool digitally. In which order will the nurse perform the steps, starting with the first one? 1. Obtain baseline vital signs. 2. Apply clean gloves and lubricate. 3. Insert index finger into the rectum. 4. Identify patient using two identifiers. 5. Place patient on left side in Sims' position. 6. Massage around the feces and work down to remove. a. 4, 1, 5, 2, 3, 6 b. 1, 4, 2, 5, 3, 6 c. 4, 1, 2, 5, 3, 6 d. 1, 4, 5, 2, 3, 6

ANS: A The steps for removing a fecal impaction are as follows: identify patient using two identifiers; obtain baseline vital signs; place on left side in Sims' position; apply clean gloves and lubricate; insert index finger into the rectum; and gently loosen the fecal mass by massaging around it and work the feces downward toward the end of the rectum.

A nurse is assessing body alignment. What is the nurse monitoring? a. The relationship of one body part to another while in different positions b. The coordinated efforts of the musculoskeletal and nervous systems c. The force that occurs in a direction to oppose movement d. The inability to move about freely

ANS: A The terms body alignment and posture are similar and refer to the positioning of the joints, tendons, ligaments, and muscles while standing, sitting, and lying. Body alignment means that the individual's center of gravity is stable. Body mechanics is a term used to describe the coordinated efforts of the musculoskeletal and nervous systems. Friction is a force that occurs in a direction to oppose movement. Immobility is the inability to move about freely.

A nurse observes a patient rising from a chair slowly by pushing on the chair arms. Which type of tension and contraction did the nurse observe? a. Eccentric tension and isotonic contraction b. Eccentric tension and isometric contraction c. Concentric tension and isotonic contraction d. Concentric tension and isometric contraction

ANS: A This movement causes eccentric tension and isotonic contraction. Eccentric tension helps control the speed and direction of movement. For example, when using an overhead trapeze, the patient slowly lowers himself to the bed. The lowering is controlled when the antagonistic muscles lengthen. By pushing on the chair arms and rising eccentric tension and isotonic contraction occurred. In concentric tension, increased muscle contraction causes muscle shortening, resulting in movement such as when a patient uses an overhead trapeze to pull up in bed. Concentric and eccentric muscle actions are necessary for active movement and therefore are referred to as dynamic or isotonic contraction. Isometric contraction (static contraction) causes an increase in muscle tension or muscle work but no shortening or active movement of the muscle (e.g., instructing the patient to tighten and relax a muscle group, as in quadriceps set exercises or pelvic floor exercises).


Related study sets

NSG252-EXAM4-Patho Book CH 16 - Disorders of Brain Function (Stroke, TBI, Seizures, etc)

View Set

6th Grade The Water Cycle and Influences on Weather and Climate Study Guide

View Set

Fact Cards Set 5 Plus 9, Minus 9 Facts

View Set

Dr. McCann Prelab on the Anatomy of the Eye

View Set

Ch 6 Practice Quiz Constitution (Legal Environment of Business)

View Set